OB/GYN ARDMS REVIEW

Pataasin ang iyong marka sa homework at exams ngayon gamit ang Quizwiz!

Right after neonatal UT

- After neonatal, the cx AP is equal to or slightly greater than the uterine fundus

What happens in conception?

- Day 14 - ovulation - Graafian follicle rupures and liberates the ovum. - Fimbria of fallopian tube transports ovum - sperm enters ovum in ampulla -cells that surround the blastocyst, scyncytiotrophoblastic cells, then begin to produce HCG - conception usually occurs within 24 hours (on day 15)

iliopsoas muscles

- Formed by psoas major and iliacus muscles - lateral and anterior to iliac crest

uterine arteries

- branches of internal iliac artery that supplies blood to the uterus, ovaries and fallopian tubes

Midline anatomy

- falx cerebri - interhemispheric fissure - corpus callosum - cavum septum pellucidum - third ventricle - Aqueduct of sylvious (cerebral aqueduct) - Fourth ventrivle - Cerebellar vermis - Cisterna Magna Interthalmic adhesion (massa intermedia)

suspensory ligament of the ovary

- pelvic ligament that provides support to the Ovary and extends from the lateral portion of the Ovary to the pelvic side walls - AKA infundibulopelvic ligament

Prepubertal UT

- prepubertal UT has a cx to UT ratio of 2:1. The uterus grows slow in prepubertal years

Uterine plexus

- situated along sides and cornua of uterus - are not tortuous like the arteries

Round ligament

- supports the Uterus (fundus) - extends from uterine cornua to labia majora between the folds of the broad ligaments - maintains anteflexion/if stretches too much can retroflex - contracts during labor

The uterosacral ligament

- supports the uterus - extends from the uterus to sacrum

Location of obturator internus

- true pelvis muscles located lateral to the ovaries - level of the vagina -posteromedial to the iliopsoas

Neonatal uterus

- tubular in appearance with different endometrial echoes in the 1st week of life due to maternal hormone stimulation

broad ligament is and supports

- wing like ligament - creates retrouterine and vesicouterine pouches - drapes over: Uterus fallopian tubes ovaries blood vessels

During the follicular phase....

-Day 14, LH is produced and stimulates ovulation, graffian follicles rupture and expels some fluid and ovum into peritoneum -Ovum is picked up by fimbria and is propelled through the tube to then be fertalized or reabsorbed or passed with menstruation.

When does follicular phase start?

-On day 1 till day 14 and ending with ovulation.

The suspensory ligament of the ovaries (infundibulopelvic) supports and contains

-Ovaries and tubes -contains ovarian arteries ovarian veins lymphatics ovarian nerves -extends from ovaries to pelvic side walls

Graafian follicle

-The name for the dominant follicle prior to ovulation -Contains the Egg (oocyte) within a region called cumulus oophorus

ovarian ligament

-extends from the cornua of the uterus to the medial aspect of the ovary

levator ani muscles

-forms the pelvic floor along with the piriformis muscle - posteroinferior to the vagina and cervix - Most caudal structures in the pelvic cavity - medial to obturator internus mucles - hammock-shaped pelvic muscle

During Luteal phase

-graafian follicle ruptures and turns into an endocrine gland called the corpus luteum -corpus luteum primarily creates progesterone to maintain thickness of endo and prepare the endo for a fertalized ovum. and small amount of estrogen. All other follicles undergo atresia.

Estrogen replacement therapy reduces and increases what risks?

-reduces the risk of Alzheimers, CHD, colon ca and osteoporosis. -increases the risk of Breast ca, endometrial carcinoma, DM, HTN, and thombus. progesterone therapy is added to reduce risks.

Ovarian arteries arise from

-the lateral margins of the abd ao, slightly inferior to the renal arteries.

postpubertal UT

-uterus grows bigger than cx -multigravida UT is longer

Sonographic findings of Agenesis of Corpus Callosum and Cavum Septum pellucidum

1. "Sunburst" sign-radial arrangement of the sulci 2. Colpocephaly- tear drop 3. elevated and dilated 3rd ventricle

Sonographic findings of radial ray defect

1. Absent or hypoplastic radius 2. Various defects in other body systems: cardiac and VACTERL association

Sonographic findings of caudal regression syndrome

1. Absent sacrum (sacral agenesis) and possibly part of the lumbar vertebra 2. Possible abnormalities in the lower extremities like clubfeet.

What are the clinical findings of granulosa cell tumors?

1. Adolescent - pseudoprecocious puberty 2. Reproductive ages and postmenopausal women - abnormal vaginal bleeding

Complete (spontaneous) abortion

1. All products of conception expelled 2. prominent endometrium, which may contain hemorrhage. 3. Abortion before 20 weeks gestation

Sonographic findings of amniotic band syndrome

1. Amputation of fetal parts or severe edema in the affected area 2. facial clefting

Sonographic findings of Vein of Galen Aneurysm

1. Anechoic mass within the midline of the brain that contains turbulent arterial and venous flow when interrogated with pulsed and color doppler 2. Fetal hydrops 3. cardiomegaly ( caused by cardiac overload)

Sonographic appearance of Choroid plexus cyst

1. Anechoic, round, smooth-walled cysts located within choroid plexus of the lateral ventricles ( regresses by end of 3rd trimester are assoc. w/ tri 18) measure more than 2mm

Sonographic findings of OHS

1. Ascites and possible pleural effusion 2. Cystic enlargement of the ovaries >5cm 3. theca lutein cysts that can cause Pain and torsion

What are the clinical findings of endometrioma?

1. Asymptomatic 2. pelvic pain 3. infertility 4. dysmenorrhea 5. menorrhagia 6. despareunia 7. painful bowel movements

Sonographic findings of aqueductal stenosis

1. Atrium of the lateral ventricle meas >10mm 2. Atrial measurement >15mm is considered moderate to marked ventriculomegaly 3. Dangling choroid sign 4. Dilatation of the lateral ventricles and the third ventricle; the fourth ventricle remains normal.

Sonographic findings of ventriculomegaly

1. Atrium of the lateral ventricle measures >10mm 2. Atrial measurement >15mm is considered 3. Dangling choroid sign 4. Dilatation of any part of the ventricular system

Fitz Hugh Curtis Syndrome

1. Chlamydia and Gonorrhea lead to Perihepatic Infection 2. Adhesions between the liver and diaphragm 3. Liver capsule becomes inflamed, mimicing gallbladder disease 4. Elevated liver function tests 5. free fluid

What are the clinical findings of ectopic pregnancy?

1. Classic clinical Triad: pain/vaginal bleeding, palpable abdominal/pelvic Mass 2. shoulder pain secondary to intraperitoneal Hemorrhage with diaphragmatic irritation 3. cervical tenderness

Sonographic findings of sacrococcugeal teratoma

1. Complex mass extending off of the distal fetal spine 2. Mass can be highly vascular 3. Hydronephrosis may be present (when mass invades the pelvis) 4. Fetal hydrops may be present

What are the sonography findings of a cystic teratoma?

1. Complex, partially cystic mass in the ovary that includes one or more academic structures that make Shadow 2. tip of the iceberg sign 3. dermoid plug produces posterior shadowing 4. dermoid mesh produced by hair and will appear as numerous linear interfaces within the cystic area of this Mass

Sonographic findings of subchorionic hemorrhage

1. Crescent-shaped anechoic, echogenic, or hypoechoic area adjacent to the gest sac (depends upon the age of the hemorrhage) 2. May resemble a second gest sac

Facial anomalies of Alobar holoprosencephaly

1. Cyclopia 2. Hypotelorism 3. Proboscis 4. (Median) cleft lip/palate 5. Anopthalmia 6. Cebocephaly 7. ethmocephaly

Sonographic findings of Porencephaly

1. Cystic mass that communicates with lateral ventricles 2. most often unilateral ( Arachnoid cyst will not communicate w/ ventricular system)

What is the most common and 2nd most common malignant germ cell tumor of the ovary?

1. Dysgerminoma 2. Yolk sac Tumor

Sonographic findings of Mega Cisterna Magna

1. Enlargement of the Cisterna Magna >10 mm in (AP) dimension 2. Normal Cerebellum and 4th ventricle

Sonographic findings of dandy-walker malformation

1. Enlargement of the Cisterna Magna >10 mm in the AP dimentsion 2. Agenesis or Hypoplasia of the Cerebellar Vermis 3. Varying degrees of ventriculomegaly

Clinical findings of OHS Ovarian Hyperstimulation syndrome

1. Fertility treatment, including ovulation induction 2. Electrolyte imbalance 3. Oliguria 4. N/V 5. Abdominal distension 7. can initiate renal failure, thrombus, and acute respiratory distress syndrome

Sonographic findings of Schizencephaly

1. Fluid filled clefts within the cerebrum 2. Agenesis of the Cavum Septum Pellucidum ( 50%) 3. Ventriculomegaly left image abnormal

Sonographic findings of hydranencephaly

1. Fluid-filled cranium 2. Absent or partial abscence of the falx cerebri 3. Maintained brain stem and basal ganglia 4. No identifiable cerebral cortex

The Clinical Findings of Endometritis

1. HX of Abortion, Postpartum, D&C, PID, Surgery, and IUD 2. Pelvic Tenderness 3. Fever 4. Leukocytosis

Sonographic findings of alobar holoprosencephaly

1. Horseshoe shaped monoventricle 2. Fused echogenic thalami 3. Absence of multiple structures 4. Normal cerebellum and brain stem

What are the clinical findings of a complete molar pregnancy?

1. Hyperemesis gravidarum 3. vaginal bleeding 4. enlarged uterus 5. possible preeclampsia or eclampsia 6. hypertension

Sonographic findings of a Uterine Leiomyoma/Leiomyosarcoma

1. Hypoechoic mass within the UT 2. Posterior shadowing from the mass 3. Degenerating fibroids may have calcifications or cystic components 4. Multiple fibroids appear as enlarged, irregularly shaped, diffusely heterogeneous UT.

Sonographic findings of Adenomyosis

1. Hypoechoic or echogenic areas adjacent to endo 2. Heterogeneous myometrium with myometrial cysts 3. Ill-defined interface between myometrium and endometrium 4. Thickening of the posterior myometrium

What are the sonography findings of complete molar pregnancy?

1. Large complex Mass within the uterus 2. vesicular snowstorm appearance secondary to placental enlargement 3. multiple variable sized cyst replacing the placental tissue 4. bilateral ovarian theca lutein cyst

Sonographic Findings of Blighted Ovum

1. Large, irregular, gest sac without an embryo or yolk sac 2. Absent or minimal gestational sac growth 3. Poor decidual reaction

What are the clinical findings of a thecoma/fibroma/brennor?

1. MEIGE SYNDROME (ASCITES AND PLEURAL EFFUSION) 2. May be asymptomatic 3. vaginal bleeding due to estrogen(thecoma only) 4. palpable mass or pain (brennor tumor only)

sonographic findings of achondroplasia

1. Macrocrania 2. frontal bossing 3. flattened nasal bridge 4. micromelia-shortening of all limbs (resulting from rhizomelia - shortening of proximal limb) 5. Trident hand

Clinical findings of adenomyosis

1. Multiparous 2. Uterine enlargement 3. Menometrorrhea

Sonographic findings of embryonic or fetal demise

1. No detectable fetal heart activity in a pole that meas 4-5mm 2. Irregularly shaped fetus 3. Irregularly sized gest sac 4. Irregular-appearing yolk sac (misshaped, calcified, or echogenic)

Sonographic findings Arnold-Chiari 2 malformation

1. Obliterated Cisterna Magna 2. Banana-shaped cerebellum 3. lemon shaped skull 4. Colpocephaly 5. Enlarged Massa Intermedia 6. Hydrocephalus 7. Open Spinal defect

What are the clinical findings of a cystic teratoma?

1. Often asymptomatic 2. if torsion or rupture occurs the patient may present with acute pelvic pain

Incomplete abortion

1. Part of the products of conception expelled 2. No detectable fetal heart motion detected 3. abnormal fetal shape

Clinical findings of uterine leiomyoma and leiomyosarcoma

1. Pelvic pressure and pelvic mass 2. Menorrhagia 3. Urinary frequency 4. Painful urination/defecation 5. Constipation 6. Infertility

Clinical findings of vaginal obstructions

1. Pelvic/Abd Pain (often at the time of menses) 2. Enlarged UT 3. Urinary retention 4. Amenorrhea (adolescent girls)

What are the sonography findings of corpus luteum of pregnancy?

1. Simple cyst appearance 2. hemorrhagic cyst variable appearances including complex components or entirely echogenic depending on the amount of blood and stage of lysis

Sonographic Findings of Acute PID

1. Thickened Irregular Endometrium 2. Ill defined uterine borders 3. Pyosalpinx 4. Hydrosalpinx 5. Cul de sac Fluid 6. Multicystic and solid complex andexal masses (tubo-ovarian complex or abscess)

The Sonographic Findings of Endometritis

1. Thickened echogenic or irregular endometrium 2. Endometrial Fluid 3. Ring down artifact (Gas or air within the endometrium)

Clinical findings of Blighted Ovum

1. Vaginal Bleeding 2. Reduction of pregnancy symptoms 3. Low HCG

Clinical findings of embryonic or fetal demise

1. Vaginal bleeding 2. small for dates 3. closed cervix 4. Low (based on LMP) HCG

Threatened abortion

1. Vaginal bleeding before 20 weeks gestation; closed cervical os 2. low fetal HR

What are the clinical findings of theca lutein cyst?

1. Very High HCG as seen in cases of GTD, ovarian hyperstimulation, twin geststations 2. N/V 3. pain associated with hemorrhagic, rupture, and ovarian torsion

What are the sonography findings of ectopic pregnancy?

1. adnexal ring sign 2. adnexal Mass 3. large amount of free fluid within the pelvis or in Morrison pouch 4. complex free fluid could represent hemoperitoneum 6. poor decidual reaction

clincial findings of endo polyps

1. asymptomatic 2. menometrorrhagia 3. intermenstrual bleeding 4. infertility in reproductive ages

sono findings of Asherman syndrome

1. bright areas within the endo 2. sonohysterography findings include bright bands of tissue traversing the uterine cavity

Sonographic findings of thanatophoric dysplasia

1. coverleaf skull 2. hydrocephalus 3. depressed nasal bridge 4. bell-shaped chest (narrow thorax) 5. polyhydramnios 6. redundant soft tissue 7. lung hypoplasia due to small chest

Sonographic findings of osteogenesis imperfecta

1. demineralization of the skull (trdcr pressure can alter the shape of the skull) 2. Multiple fractures 3. Bell-shaped chest.

What are the sonography findings of ovarian torsion?

1. enlarged ovary in the presence of multi follicular development

sono findings of endo polyps

1. focal thickening of the endo 2. diffuse thickening of the endo

Sonographic findings of sirenomelia

1. fusion of the lower extremities 2. bilateral renal agenesis 3. oligohydramnios (possibly anhydramnios)

clinical findings of Asherman syndrome

1. history of d&c, trauma, uterine surgery 2. recurrent preg loss 3. amenorrhea or hypomenorrhea

Clinical findings of spina bifida occulta.

1. in utero- normal laboratory values 2. postnatal - sacral dimple, hemangioma, lipoma, or excessive hair is identified directly over the distal spine.

Sonographic findings of Lissencephaly " smooth-brain"

1. lack of sulci and gyri within cerebrum ( not typically diagnosed until 3rd trimester)

Sonographic findings of clubfoot (talipes equinovarus)

1. metatarsals lie in the same plane as the tibula and fibula.

What are the contributing factors for ectopic pregnancy?

1. multiparity 2. Advanced maternal age 3. previous tubal ligation 4. Hx of PID 5. undergoing infertility treatment 6. previous or present use of intrauterine contraceptive device

Sonographic findings of cephaloceles

1. open cranial defect (typically posterior in location) 2. Small or obliterated cisterna magna 3. Complex mass protruding from the cranium

clinical findings of endometrial carcinoma

1. postmenopausal bleeding 2. intermenstrual bleeding 3. enlarged uterus 4. elevation of CA-125

clinical findings of endometrial atrophy

1. postmenopausal vag bleeding

Sonographic findings of achondrogenesis

1. severely shortened libs (micromelia) 2. Absent mineralization of the skull, spine, pelvis, and limbs 3. polyhydramnios

Sonographic findings of limb-body wall complex

1. short or absent umbilical cord 2. marked scoliosis 3. ventral wall defects 4. limb defects 5. craniofacial defects (exencephaly or encephalocele) 6. elevated MSAFP due to open ventral wall

Sonographic findings of spina bifida aperta (cystica)

1. splaying of laminae in the area of the defect 2. cystic mass (meningocele) or complex mass (myelomeningocele) protruding from the spine 3. lemon spine - lemon shaped cranium with flattened frontal bones 4. banana sign - banana shaped cerebellum 5. obliterated cisterna magna 6. colpocephaly 7. hydrocephaly

clinical findings of endometrial hyperplasia

1. tamoxifen therapy 2. abnormal uterine bleeding (any age) 3. PCOS 4. obesity

sono findings of endometrial hyoerplasia

1. thickened echogenic endo (>8mm) 2. small cystic areas within the endo

sono findings of endo carcinoma

1. thickened endometrium 2. heterogeneous uterus 3. enlarged uterus with lobular contour 4. endometrial fluid 5. polypoid mass within the endo

sono findings of endometrial atrophy

1. thin endo <5mm 2. intracavity fluid may be seen

How to get abd circumference

1. umbilical vein 2. fetal GB 3. thoracic spine 4. stomach 5. left adrenal gland

Clinical findings of caudal regression syndrome

1. uncontrolled maternal diabetes

Clinical findings of sirenomelia

1. uncontrolled maternal diabetes

Clinical findings of subchorionic hemorrhage

1. vaginal bleeding 2. Uterine cramping 3. closed cx

Inevitable abortion

1. vaginal bleeding with dilated cx 2. low-lying gest sac 3. open internal os of cx

Sonographic findings of a cystic hygroma

1.cystic neck mass divided in the midline by a thick fibrous band of tissue 2. the mass may contain smaller cystic areas with internal septations

The second trimester typically refers to weeks: A. 12 thru 25 B. 13 thru 26 C. 10 thru 28 D. 26 thru 42

13 through 26

At what day of the menstrual cycle does ovulation occur as the dominant follicle ruptures releasing the mature ovum and a small amount of follicular fluid into the peritoneal cavity?

14

The fetal stomach should be visualized by: 8 wks 14 wks 20 wks 18 wks

14 wks

When does ovulation occur?

14th day cause by a surge of LH

NF are meas in what wks of gest

15-19 wk

Triple screen is performed between

15-20 weeks

The cavum septum pellucidum and corpus callosum is completely formed by

18 weeks

Gest sac grows how much per day in early pregnancy?

1mm

The cerebellum grows at a rate of

1mm per week between 14 and 21 weeks and correlates close to gestational age of fetus. ex. 16wk fetus will meas 16mm

Phases of the ovarian cycle

1st - Follicular Phase 2nd - Luteal Phase

The embryonic heart begins as: 2 tubes 4 tubes 8 folds 1 tube

2 tubes

Corpus luteum meas

2-3cm sometimes up to 10cm and regresses near the first trimester

In adenomyosis, the basal layer of the endo can extend into the myometrium at depths of at least

2.5mm

HCG can be detected on a pregnant patient by what day?

23 days

rhizomelia occurs after what wks of gest

24 weeks.

Types of skeletal dysplasias and the most common

271 types. Most common: achondroplasia anchondrogenesis osteogenesis imperfecta thanatophoric dysplasia

The 4th vent has how many apertures through which the CSF travels

3 2 lateral apertures (Foramen of Luschka) 1 medial aperture (Foramen of Magendie)

How many layers does the fallopian tube have and what are they?

3, outer serosa, middle muscular layer, inner mucosal layer

The Clinical Findings of Endometriosis

3. Infertility 6. Menorrhagia 7. Painful bowel movement

BPP exam lasts

30 min

The ventricular system is composed of

4 ventricles whose primary function is to provide cushioning for the brain

Gest sac forms what weeks

4-5

Embryo forms in what weeks

5-6

Secondary yolk sac/chorionic and amnionic cavities forms what weeks

5.5 weeks

Peak incidence of endometrial carcinoma is

50-65 yrs of age

The average age at which menopause occurs

51

What are the clinical findings of corpus luteum of pregnancy?

1. Asymptomatic 2. pain associated with Hemorrhage and enlargement of cyst

What options can you do to treat ectopic pregnancies?

1. Methotrexate when confined to fallopian tube 2. potassium chloride when in cervix or cornua

PCOS ovarian volume should not exceed?

10 ml

The cisterna magna should not exceed what measurement in the transcerebellar plane? A. 4 mm B. 2 mm C. 8 mm D. 10 mm

10 mm

The fetal heart is fully formed by: 2 wks 4 wks 8 wks 10 wks

10 wks

Cisterna magna should not meas more than

10mm or less than 2mm

NT is measures between

11-14 weeks

Early 1st trimester screening consists of

11-14 weeks HCG levels pregancy-associated plasma protein A NT meas with sonography

NT are meas in what weeks of gest

11-14 wks

The palate closes by

12 weeks

The thickness for the nuchal fold in the 2nd trimester should not exceed: 3mm 6mm 10mm 12mm

6mm

appendicular skeleton forms in what wks of gest

6th-8th wk

Fetal limb/buds are identified by

7 weeks

The fetal lip closes between what weeks

7-8 weeks

Sperm lasts how long?

72 hours

How many bones, types and locations of cranial bones

8 Frontal bone - anterior Parietal (2) - superolateral temporal (2) - inferolateral Occipital - posterior Sphenoid - lateral Ethmoid - anterior between orbits

Highest BPP points

8 points w/o nonstress test. 10 points with nonstress test

Rhombancephalon (hindbrain) is seen in what weeks

8 week

The fetal lip typically closes by: 18 wks 8 wks 13 wks 6 wks

8 wks

Normal lat vent meas

<10mm

Advanced maternal age is considered to be: A. > 25 years of age B. > 30 years of age C. > 35 years of age D. >40 years of age

>35 years of age

Which of the following laboratory test may be used as a tumor marker for an ovarian dysgerminoma? A. Lactate dehydrogenase B. AFP C. CA-125 D. Tamoxifen

A

Subchorionic hemorrhage

A bleed between the endometrium and gestational SAC at the edge of the placenta

Implantation bleeding

A bleed that occurs at the time in which the conceptus and plants in the decidualized endometrium

Schizencephaly

A cerebal malformation associated with the development of fluid-filled clefts

Methotrexate

A chemotherapy drug used to attack rapidly dividing cells like those seen in an early pregnancy this drug is often used to manage ectopic pregnancies

Inhibin a

A peptide hormone secreted by the placenta during pregnancy

Cistern

A prominent space within the school that contains cerebrospinal fluid, a cistern is created by the separation of the arachnoid membrane and Pia Mater

Medial cleft lip

A subdivision within the midline of the lip

Blood accumulation within the uterus is termed: A. Hematometra B. Hydrometra C. Asherman syndrome D. Endometrial carcinoma

A. Hematometra

A 55 year old patient presents to the ultrasound department with a history of pelvic pressure, abdominal swelling, and abnormal uterine bleeding. A pelvic sonogram reveals a large, multilocualted cystic mass with papillary projections. What is the most likely diagnosis? A. Serous cystadenocarcinoma B. cystic teratoma C. androblastoma D. dysgerminoma

A. serous cystadenocarcinoma

The type of renal cystic disease associated w/ adult liver and pancreatic cysts is: MCDK ARPKD ADPKD VATER

ADPKD

A protein produced by the yolk sac and fetal liver that's found in excess in the maternal circulation in the presence of a neural tube defect is: PAAP-A hCG Estriol AFP

AFP

What does the yolk sac produce?

AFP and plays an important role in angiogenesis and hematopoiesis during early development.

atrioventricular defect

AKA endocardial cushion defect, abnormal development of the central portion of the heart

Rockerbottom feet

Abnormal curved shape of the sole of the feet

Achondroplasia is associated w/ all of the following except: Frontal bossing Flattened nasal bridge Trident hand Absent mineralization of the skull

Absent mineralization of the skull

A disorder that results in abnormal bone growth and dwarfism is: *Osteogenesis imperfecta *Achondroplasia *Radial ray defect *Caudal regression syndrome

Achondroplasia

The most common nonlethal skeletal dysplasia is: Achondrogenesis Achondroplasia Thanatophoric dysplasia Osteogenesis imperfecta

Achondroplasia

What are the clinical findings of ovarian torsion

Acute unilateral abdominal or pelvic pain 2. nausea and vomiting

Focal adenomyosis is termed

Adenomyoma

Dysmenorrhea GYN diagnosis

Adenomyosis Endometriosis

Pelvic pressure and/or tenderness GYN diagnosis

Adenomyosis Endometriosis Leiomyoma Leiomyosarcoma Ovarian mucinous cystadenocarcinoma Ovarian mucinous cystadenoma Ovarian serous cystadenocarcinoma Ovarian serous cystadenoma Pediatric - hydrocolpos or hematocolpos Pelvic inflammatory disease

Dyspareunia GYN diagnosis

Adenomyosis Endometriosis Pelvic inflammatory disease

Chronic pelvic pain GYN Diagnosis

Adenomyosis Endometriosis (endometrioma) Leiomyoma (fibroid) Pelvic inflammatory disease

Synechiae

Adhesions

What can coexist with dandy walker malformation?

Agenesis of corpus callosum ventriculomegaly holoprosencephaly cephaloceles

The "sunburst" of the cerebral sulk is a sonographic finding of: A. Dandy-walker malformation B. agenesis of the corpus callosum C. colpocephaly D. hydranencephaly

Agenesis of the corpus callosum

An omphalocele may contain: Fetal liver Ascites Fetal colon All of the above

All of the above

Cleft lip and cleft palate may exist w/: *Amniotic band syndrome *Holoprosencephaly *Trisomy 13 *All of the above

All of the above

Which of the following best describes hypospadias? *OEIS complex in the presence of a hydrocele *The chronic obstruction of the renal pelvis and urethra *The underdevelopment of the scrotum in the presence of a hydrocele *An abnormal ventral curvature of the penis

An abnormal ventral curvature of the penis

TORCH infections

An acronym that stands for toxoplasmosis, other infections, rubella, cytomegalovirus, herpes simplex virus, this group of infections may be acquired by a woman during pregnancy

Fetal karyotyping

An analysis of fetal chromosomes

Aperture

An opening in a structure

Omphalocele

And anterior abdominal wall defect where there is a herniation of the fetal bowel and other abdominal organ into the base of the umbilical cord

Pregnant patients with RLQ pain may have

Appendicitis

A lemon headshape is associated with

Arnold chiari II malformation

The form of inheritance in which at least one parent has to be a carrier of an abnormal gene for it to be passed to the fetus is: Autosomal recessive Autosomal dominant Inherited dominant Inherited recessive

Autosomal dominant

The renal cystic disease that results in the development of cysts late in adulthood is: *Multicystic dysplastic renal disease *Autosomal dominant polycystic disease *Autosomal recessive polycystic disease *Obstructive cystic dysplasia

Autosomal dominant polycystic disease

Fluid noted posterior to the uterus would most likely be located within the: A. Space of Retzius B. Pouch of Douglas C. Anterior cul-de-sac D. Adnexa

B

Which term below is best defined as excessive bleeding from the uterus in duration and amount? A. Dysmenorrhagia B. Menorrhagia C. Menometrorrhagia D. Netrorrhagia

B

Ovulation typically occurs on day ______ of the menstrual cycle. A. 12 B. 14 C. 16 D. 1

B. 14

The longest and most tortuous segment of the fallopian tube is the: A. fimbria B. ampulla C. Isthmus D. interstitial

B. ampulla

Macroglossia is most commonly found w/: Anencephaly Holoprosencephaly Beckwith-Wiederman syndrome Cystic hygroma

Beckwith-Wiederman syndrome

Hepatomegaly would be seen in conjunction w/: Down syndrome Edward syndrome Beckwith-Wiedermann syndrome Hirschsprung disease

Beckwith-Wiedermann syndrome

What is the most common association with macroglossia

Beckwith-wiedemann syndrome and down syndrome

Meigs syndrome GYN diagnosis

Brennor tumor Fibroma (most often) Thecoma

Which of the following is defined as pain during intercourse? A. Dysuris B. Dysmenorrhea C. Dyspareunia D. Hirsutism

C

The cervix should measure at least: A. 4cm B. 5cm C. 3cm D. 8cm

C. 3cm

The measurement of the endometrium during the proliferative phase ranges from: A. 6 to 10 mm B. 8 to 12 mm C. 4 to 8 mm D. 1 to 2 mm

C. 4 to 8 mm

The most common placental tumor is the: A. Choriocarcinoma B. Maternal lake C. Choriangioma D. Allantoic cyst

C. Choriangioma

In VACTERL association, the letter "C" stands for: Cerebellar C-spine Cranial Cardiac

Cardiac

The structure located between the two lobes of the cerebellum is the: A. cerebellar vermis B. cerebellar tonsils C. falx cerebri D. corpus callosum

Cerebellar vermis

WHat is the most common female malignancy in women younger than 50

Cervical carcinoma

Secondary amenorrhea

Characteristically diagnosed in the postmenopausal women who has had at least 12 months without a menstrual cycle

The bending of the fifth digit toward the fourth digit is called: A. Syndactyly B. clinodactyly C. polydactyly D. Stabodactyly

Clinodactyly

OEIS complex is also referred to as: Bladder exstrophy Omphalocele Potter syndrome Cloacal exstrophy

Cloacal exstrophy

The band of tissue that allows communication between the right and left cerebral hemispheres is the: A. Falx cerebri B. corpus callosum C. cerebellar vermis D. cavum septum pellucidum

Corpus callosum

The physiologic ovarian cyst that develops after ovulation has occurred is the: A. Theca internal cyst B. Graafian cyst C. Corpus Luteum Cyst D. Cystic teratoma

Corpus luteum cyst

A Dolichocephaly headshape is associated with

Craniosyntosis

Fusion of the orbits is termed: Microglossia Cebocephaly Cyclopia Ethmocephaly

Cyclopia

All of the following are sonographic features of holoprosencephaly except: Cystic hydroma Proboscis w/ cyclopia Fused thalamus Monoventricle

Cystic hydroma

A large, mostly cystic mass is noted in the cervical spine region of a fetus. This most likely represents a: *Sacrococcygeal teratoma *Cystic teratoma *Cephalocele *Anophthalmia

Cystic teratoma

A lemon-shaped skull is related to: A. spina bifida B. Arnold-chiari II malformation C. Both a and b D. all of the above

D

Which of the following would not be decreased in the presence of Edwards syndrome? A. Estriol B. hcg C. alpha-fetoprotein D. All would be decreased

D

Treatment of GTD

D and C, HCG monitoring, hysterectomy, chemotherapy.

The average menstrual cycle lasts: A. 45 days B. 24 days C. 26 days D. 28 days

D. 28 days

The sonographic evidence of a hyperemic fallopian tube is consistent with: A. Pyosalpinx B. Hydrosalpinx C. Endometritis D. Salpingitis

D. Salpingitis

The arteries within the functional layer of the endometrium that are altered by the hormones of the ovary are the: A. Arcuate arteries B. Radial arteries C. Straight ovaries D. Spiral arteries

D. Spiral arteries

All of the following are clinical findings with endometrial hyperplasia except: A. Obesity B. Polycystic Ovary Syndrome C. Abnormal uterine bleeding D. Thickened endometrium

D. Thickened endometrium

The visualization of the fetal stomach w/in the fetal chest is most indicative of: *Pulmonary sequestration *Diaphragmatic hernia *Turner syndrome *Cystic adenomatoid malformation

Diaphragmatic hernia

Pyelectasis refers to: *Enlargement of the urinary bladder, ureter, and renal calices *Dilation of the ureter *Dilation of the renal pelvis *Enlargement of the ureter only

Dilation of the renal pelvis

What is shown to be effective at evaluating hypoxia?

Doppler of the mid cerebral artery.

All of the following are sonographic features of alobar holoprosencephaly except: A. cyclopia B. monoventricle C. dorsal cyst D. fused thalamus

Dorsal cyst

Pathologies with Low Estriol/Low MSAFP/low PAPP-a

Down Syndrome (T21) Edwards syndrome (T18) Turner syndrome

Pathologies with High inhibin A

Down Syndrome (T21) Patau syndrome (T13)

A 38 year old pregnant woman presents to the sonography department for an obstetrical sonogram with abnormal maternal serum screening. Her Alpha fetoprotein and estriol are low, while her human chorionic gonadotropin is elevated. These laboratory findings are most consistent with: a. Edwards syndrome b. Patau syndrome c. Triploidy d. Down syndrome

Down syndrome

Brachycephaly is associated most often with the following syndromes? A. Edwards syndrome B. Patau syndrome c. Down syndrome D. Turner syndrome

Down syndrome

Ectoderm Mesoderm Endoderm

Ectoderm - The outer germ cell layer of the embryo that develops into the integumentary system and central nervous system. Mesoderm - layer of the embryo that develops into the circulatory system, musculoskeletal system, reproductive system, etc. Endoderm - layer of the embryo that develops into the GI and respiratory tracts

Pathologies with Low HCG

Ectopic Down syndrome (trisomy 21) Edwards syndrome (trisomy 18) Turner syndrome

The condition in which the heart is located on the outside of the chest wall is termed: *Cystic adenomatoid malformation *Coarctation of the heart *Cardiac sequestration *Ectopic cordis

Ectopic cordis

Low Hematocrit GYN diagnosis

Ectopic pregnancy Trauma

A strawberry-shaped skull is associated with: A. Edwards syndrome B. Turner Syndrome C. Down syndrome D. Patau syndrome

Edwards syndrome

Pathologies with Low inhibin A

Edwards syndrome (T18) Turner Syndrome

The congenital absence of part of the esophagus is termed: Duodenal atresia VACTERL syndrome Down syndrome Esophageal atresia

Esophageal atresia

Estrogen vs progesterone

Estrogen - released during proliferative phase that initiates the proliferation and thickening of the endo Progesterone - prepares the uterus for pregnancy, maintains pregnancy and promotes development of the mammary glands; primarily produced by the ovary and placenta. Keeps endo thick.

The condition in which there is no nose and a proboscis separating two close-set orbits is: Ethmocephaly Epignathus Micrognathia Cebocephaly

Ethmocephaly

Hyperemesis gravidarum

Excessive vomiting during pregnancy

What organ(s) produces amniotic fluid after 12 wks? *Fetal liver and spleen *Fetal intestines and lungs *Fetal intestines and liver *Fetal kidneys

Fetal kidneys

Chorionic villi

Finger-like projections of gestational tissue that attach to the decidualized endometrium and allow transfer of nutrients from the mother to the fetus

When is the first day of the menstrual cycle ?

First day of bleeding

Something that is idiopathic is said to be: A. Caused by a functional abnormality B. related to fetal development C. from an unknown cause D. found incidentally

From an unknown cause

How does implantation of ectopic endometrial tissue result?

From passing through the fallopian tubes during menstruation or may result from scarring from surgery after a c - section

Endometriosis

Functional ectopic endometrial tissue located outside the uterus

Theca lutein cysts

Functional ovarian cysts that are found in the presence of elevated levels of human chorionic gonadotropin

Cyclopia

Fusion of the orbits

Patients with RUQ pain may have

Gallstones

All of the following are sonographic features of pentalogy of Cantrell except: Omphalocele Gastroschisis Cleft sternum Diaphragmatic defect

Gastroschisis

The most common abnormality of the fetal liver is: Gallstones Hepatocellular lymphadenopathy Cirrhosis Hepatomegaly

Hepatomegaly

Gastroschisis

Herniation of abdominal contents through a right-sided Periumbilical abdominal abdominal wall defect

Which of the following laboratoryfindings would not be consistent with trisomy 21? A. High AFP B.Low estriol C. High hCG D. Low PAPP-A

High AFP

Triploidy labs

High HCG (with molar)

Anencephaly labs

High MSAFP

Cephalocele labs

High MSAFP

Gastroschisis labs

High MSAFP

Omphalocele labs

High MSAFP

Spina Bifida (meningocele or myelomeningocele) labs

High MSAFP

The Doppler analysis of malignant ovarian masses often reveal what?

Higher diastolic flow velocities because of the abnormal vessels that are created with malignancy, these vessels often lack smooth muscle within their walls and produce a less resistive waveform pattern

A functional bowel disorder w/in the fetus that is caused by the absence of intestinal nerves is found in: Gastroschisis Beckwith-Wiederman syndrome Omphalocele Hirschsprung disease

Hirschsprung disease

All of the following are associated w/ omphalocele except: Trisomy 18 Pentalogy of Cantrell Intrauterine growth restriction Hirschsprung disease

Hirschsprung disease

Another name for pelvocaliectasis is: Caliectasis Hydrocele Hydronephrosis Pyonephrosis

Hydronephrosis

The dilation of the renal collecting system secondary to the obstruction of normal urine flow defines: A. Nephrocalcinosis B. Hydronephrosis C. Renal Calculi D. Urinary Stasis

Hydronephrosis

An increase distance b/n the orbits is referred to as: Hypotelorism Hypertelorism Anophthalmia Micrognathia

Hypertelorism

Fibroids Types

Intramural - within the myometrium Intracavitary - within the uterine cavity submucosal- adjacent or going into endometrium subserosal fibroid- grows outward and distorts the contour of the uterus Pedunculated- Grows out or inside with a stalk. Can look like adnexal mass

Adhesions

Irregular bands of tissue

What does cilia in the fallopian tube do?

It creates peristalsis to move things to where it needs to go

WHen do HCG levels rise and go down?

It rises in the first trimester plateaus in second trim and decreasing with advancing gestation.

What are the clinical findings of a krukenburg tumor?

Krukenberg 1. Asymptomatic 2. weight loss 3. pelvic pain

Hydronephrosis is common during

Late pregnancy

A Structural abnormality that results from an abnormal development describes: A. Syndrome B.chromosomal deviation C. Malformation D. Congenital misrepresentation

Malformation

Dandy-Walker malformation/complex

Malformation - Congenital brain malformation in which there is an enlargement of the cisterna magna, agenesis of cerebellar vermis and dilation of fourth ventricle Complex - spectrum of posterior fossa abnormalities including the ones above

Sertoli leydig cell tumor

Malignant sex cord stromal ovarian neoplasm that is associated with virilization

Dermoid mesh

Mass of hair within a cystic teratoma

Myelomeningocele

Mass that result from the Spina Bifida that contains the spinal cord and meninges

The thalamic tissue located w/in the 3rd ventricle of the brain that can become enlarged w/ Arnold-Chiari II malformation is: Corpus callosum Cerebellar vermis Cavum septum pellucidum Massa intermedia

Massa intermedia

Sirenomelia is commonly referred to as: Radial ray defect Rhizomelia Mermaid syndrome Rockerbottom feet

Mermaid syndrome

Which of the following would be best defined as irregular menstrual bleeding between periods? A. Menometrorrhagia B. Menorrhagia C. Metrorrhagia D. Hypomenorrhea

Metrorrhagia

Monoventricle

Microcephaly one large ventricle within the brain associated with holoprosencephaly

A small mandible is termed: Macroglossia Epignathus Micrognathia Ethmocephaly

Micrognathia

The term for small eyes is : A. Microphthalmia B. Micrognathia C. Microcephaly D. Microglossia

Microphthalmia

Once the zygote undergoes rapid cellular division and eventually forms a cluster of cells it is then called

Morula

Which is larger serous or mucinous cystadenoma?

Mucinous

What is the most common cyst found in the pelvis

Nabothian cysts

All of the following are sonographic signs of Ebstein anomaly except: *Enlarged right atrium *Fetal hydrops *Narrowing of the aortic arch *Malpositioned tricuspid valve

Narrowing of the aortic arch

The most common location of a cystic hygroma is w/in the: Axilla Neck Chest Groin

Neck

The most common malignant adrenal pediatric tumor is the: A. Nephroblastoma B. Pheochromocytoma C. Hepatoblastoma D. Neuroblastoma

Neuroblastoma

Mesocephalic

Normal head shape

Women who are undergoing ovulation induction by means of hormone administration are at an increased risk for developing what?

Ovarian Hyperstimulation Syndrome

Follicular cyst

Ovarian cyst that forms from the failure of the graafian follicle to ovulate

Precotious Puberty GYN diagnosis

Ovarian dysgerminoma Ovarian granulosa cell tumor

Hormone extracted from the urine of postmenopausal women to be used for trying to have babies

Pergonal

Chlamydia or Gonorrhea can lead to what?

Perihepatic Infections (development of adhesions located between the liver and diaphragm)

Sonographically, you visualize a mass extending from the distal spine of a fetus. This mass could be all of the following except: Sacrococcygeal teratoma Meningocele Meningomyelocele Phocomeningocele

Phocomeningocele

The herniation of the bowel into the base of the umbilical cord before 12 wks is termed: Gastroschisis Omphalocele Hernia umbilicus Physiologic herniation

Physiologic herniation

Painless second trimester vaginal bleeding is most often associated with A. Placental Abruption B. Ectopic Pregnancy C. Miscarriage D. Placenta previa

Placenta previa

The premature separation of the placenta from the uterine wall before the birth of the fetus describes: A. Placenta Previa B. Placental Abruption C. Ectopic Cordis D. Subchorionic Harmatoma

Placental abruption

The sonographic "bat wing" sign is indicative of: Pericardial effusion Pulmonary atresia Pleural effusion Endocardial cushion defects

Pleural effusion

Endometritis may occur after?

Pospartum, D&C (dilatation and curettage), and IUD

prosencephalon, mesencephalon, rhombencephalon

Prosencephalon (forebrain) - becomes the lateral ventricles, cerebral hemispheres, third ventricle, thalamus, hypothalamus , pineal gland, pituitary gland Mesencephalon (midbrain) - Meningocele that becomes the cerebral peduncles,quadrigeminal plate and cerebral aqueduct Rhombencephalon (hindbrain) - becomes the cerebellum, pons, medulla oblongata, fourth ventricle

The clinical manifestations of supine hypotensive syndrome include all of the following except: A. Proteinuria B. Tachycardia C. Nausea D. Pallor

Protenuria

Ovarian torsion often occurs on which side? What is the most common cause?

Right; ovarian cyst or mass such as the benign cystic teratoma or paraovarian cyst

Brachycephalic

Round skull shape

All of the following are signs of Arnold-Chiari II syndrome except: S-shaped spine Banana sign Lemon sign Colpocephaly

S-shaped spine

sacrococcygeal teratoma vs spina bifida aperta

SCT has a normal head and intracranial anatomy while spina bifida aperta does not.

The spine is imaged in what scan planes

Sag, TRV, Coronal

How to screen for neural tube defects?

Sonography Amniocentesis maternal serum screeening

Choroid plexus

Specialized cells within the ventricular system responsible for cerebrospinal fluid production

Corpus luteum

Temporary endocrine gland that results from the rupture of the graafian follicle after ovulation

Embryo

Term given to the developing fetus before 10 weeks gestation

A cloverleaf headshape is associated with

Thanatophoric dysplasia

Neural plate

The early embryo logic structure that develops into the central nervous system

What happens days 1 to 5 of the menstrual cycle?

The endometrium sheds

Lobar holoprosencephaly

The least severe form of holoprosencephaly

Discriminatory Zone

The level of human chorionic gonadotropin Beyond which and intrauterine pregnancy is consistently visible

Ependyma

The lining of the ventricles within the brain

Proliferation

The multiplication of similar forms

Physiologic bowel herniation

The normal developmental stage when the mid gut migrates into the base of the umbilical cord

Which statement is true concerning fetal outflow tracts? *The normal pulmonary artery should be positioned posterior to the aorta and should be visualized passing under it. *The normal pulmonary artery should be positioned anterior to the aorta and should be visualized crossing over it. *The right ventricular outflow tract leads to the aorta. *The left ventricular outflow tract leads to the pulmonary artery.

The normal pulmonary artery should be positioned anterior to the aorta and should be visualized crossing over it.

Communicating hydrocephalus

The obstruction of cerebrospinal fluid from a source outside of the ventricular system

Craniosynostosis

The premature close of the cranial sutures with subsequent Fusion of the cranial bones

Placental abruption

The premature separation of the placenta from the uterine wall before the birth of the fetus

Cerebrospinal fluid

The protective and nourishing fluid of the brain and spinal cord produced by the cells of the choroid plexus

Cephalic index

The ratio used for assessing fetal head shape

Corpus albicans

The remaining structure of the corpus luteum after it is deterioration

Chorionic cavity

The space between the chorionic sac and the amniotic sac that contains the secondary yolk Sac AKA extraembryonic coelom

What are the largest and least common of the functional cysts?

Theca lutein cysts

What are the sonography findings of a thecoma/fibroma/Brennor tumor; granulosa cell?

Thecoma/fibroma/Brennor- Hypoechoic solid Mass with posterior attenuation; may contain calcifications; similar to leiomyoma Granulosa cell- variable

The venous supply of the female pelvis

They mirror the arterial counterparts except with the left ovarian vein. Instead of returning blood to the IVC, it drains directly into the left renal vein.

Triploidy vs Triploid

Triploid - A cell having three times the normal haploid number. There are 69 chromosomes. Triploidy - A fetus that has three of every chromosome

Cleft lip, hypotelorism, and microphthalmia are all sonographic features of: a. Trisomy 21 b. Trisomy 18 c. Trisomy 13 d. Turner syndrome

Trisomy 13

Cyclops would most likely be associated with : A. Trisomy 8 B. Trisomy 21 C. Trisomy18 D. Trisomy 13

Trisomy 13

Sonographically you identify a fetus with fusion of the thalami and monoventricle. Which chromosomal abnormality would be most likely? A. trisomy 8 B. Trisomy 21 C. Trisomy 18 D. Trisomy 13

Trisomy 13

Bilateralchoroid plexus cysts, Micrognathia, and rockerbottom feet are sonographic findingsof a 27 week fetus with omphalocele. These findings are most consistent with: A. Trisomy 21 B. Trisomy 13 C. Trisomy 18 D.Triploidy

Trisomy 18

Micrognathia is a condition found in: Trisomy 21 Hydranencephaly Beckwith-Wiedermann syndrome Trisomy 18

Trisomy 18

The choroid plexus cyst could be associated with an increase risk of: A. T13 B. T4 C. Arnold-chiari II malformation D. T18

Trisomy 18

A strawberry headshape is associated with

Trisomy 18 (Edwards)

A 22-week gestation fetus with clinodactyly, an echogenic intracardiac focus, and hyperechoic bowel is noted during a screening obstetrical sonogram. These findings are most consistent with: a. Trisomy 21 b. Trisomy 13 c. Monosomy X d. Trisomy 18

Trisomy 21

Absent nasal bones in an increased nuchial measurement is most consistent with the sonographic markers for: A. Trisomy 21 B. Trisomy 13 C. Triploidy D. Trisomy 18

Trisomy 21

An echogenic intracardiac focus is often seen in cases of: Trisomy 21 Trisomy 13 Trisomy 8 Turner syndrome

Trisomy 21

Most often, __________ is associated w/ duodenal atresia. Trisomy 21 Trisomy 18 Trisomy 13 Triploidy

Trisomy 21

Of the following, which is macro glossary most often associated with ? A. Trisomy 21 B. Trisomy 18 C. Triploidy D. Turner syndrome

Trisomy 21

Fetal rhabdomyomas are associated w/ which of the following? *Tracheoesophageal fistulas *Tuberous sclerosis *Eventration of the diaphragm *Tuberculosis

Tuberous sclerosis

The progression of Tubo Ovarian Complex results in?

Tubo Ovarian Abcess

All of the following are associated w/ duodenal atresia except: Trisomy 21 Esophageal atresia VACTERL syndrome Turner syndrome

Turner syndrome

Monosomy X refers to: a. Edwards syndrome b. Patau syndrome c. Down syndrome d. Turner syndrome

Turner syndrome

Cystic hygromas are found in many syndromes like

Turner syndrome fetal hydrops aneuploidy T21 T18 T13

VACTERL

VACTERL: V: vertebral A: anal atresia C: cardiac TE: tracheo-esophageal atresia R: renal L: limb

Which of the following is considered to be the most common cardiac defect? *Hypoplastic right heart syndrome *Transposition of the great vessels *Hypoplastic left heart syndrome *Ventricular septal defect

Ventricular septal defect

The dangling choroid sign is synonymous with: A. hydranencephaly B. schizencephaly C. Acrania D. ventriculomegaly

Ventriculomegaly

What is the most common cranial abnormality?

Ventriculomegaly

Which best describes the optimal instance to take the femur length measurement? A. When the epiphyseal plates are clearly identified and the shaft is parallel to the sound beam B. When the diaphysis of the femur is parallel to the sound beam C. When the long axis of the femoral shaft is perpendicular to the sound beam D. When the femoral shaft is parallel to the sound beam

When the long axis of the femoral shaft is perpendicular to the sound beam.

Do dermoids have the capability of malignant degeneration?

Yes

gartner duct cyst

a benign cyst located within the vagina

Congenital malformations of the vagina can lead to

accumulation of fluid within the female genital tract secondary to an obstruction

tubo-ovarian complex

adhesions develop that lead to fusion of the ovaries and dilated tubes as a result to PID

The ovaries can be located

anywhere within the true pelvis excluding the anterior cul-de-sac.

Arachnoid cysts vs Porencephaly

arachnoid cysts will not communicate with the ventricular system

All of the following complications are associated with multiple gestations except: a. preterm delivery b. high birth weight c. maternal anemia d. maternal preeclampsia

b. high birth weight

A simple fluid accumulation within the vagina secondary to an imperforate hymen is: a. hydrometrocolpos b. hydrocolpos c. hematometra d. hematocolpos

b. hydrocolpos

Anechoic fluid noted distending the uterus and cervix within a pediatric patient is termed: a. hydrocolpos b. hydrometrocolpos c. hydrometra d. hematometrocolpos

b. hydrometrocolpos

The twin that will appear larger in twin-twin transfusion syndrome is the: a. donor b. recipient c. both will be the same d. both will be demised

b. recipient

Endometrium can be divided into

basal layer and functional layer

Corpus means

body of uterus

Osteoporosis

bone loss that predisposes the individual to fractures.

A cephalic index of >85 denotes

brachiocephalic shape

The sonographic examination of twins reveals a triangular extension of the placenta at the base of the membrane. This finding is indicative of: a. monochorionic monoamniotic twins b. monochorionic diamniotic twins c. dichorionic diamniotic twins d. monochorionic diamniotic twins

c. dichorionic diamniotic twins

Which of the following can occur as a result of dizygotic twinning? a. monochorionic diamniotic twins b. monochorionic monoamniotic twins c. dichorionic diamniotic twins d. all of the above

c. dichorionic diamniotic twins

The inferior portion of the cervix closest to the vagina is: a. cornu b. internal os c. external os d. inferior fornix

c. external os

The recesses of the vagina are the: a. cornu b. isthmus c. fornices d. parity

c. fornices

The layer of endometrium that is altered as a result of hormonal stimulation during menstrual cycle is the: a. myometrium b. endometrial cavity c. functional layer d. basal layer

c. functional layer

Ovulation induction drugs not only increase the likelihood of multiple gestations, but also increase the likelihood of: a. maternal diabetes b. ovarian prolapse c. heterotopic pregnancies d. choriocarcinoma

c. heterotopic pregnancies

The superior portion of the cervix is the: a. cornu b. corpus c. internal os d. external os

c. internal os

The surgical removal of a fibroid is termed: a. hysterosonogram b. total abdominal hysterectomy c. myomectomy d. uterine artery embolization

c. myomectomy

Uterine veins go through what ligament

cardinal ligament

22. All of the following are clinical features of an ectopic pregnancy except: a. Pain b. Vaginal bleeding c. Shoulder pain d. Adnexal ring

d. Adnexal ring

The area of attachment of the Fallopian tubes within the uterus is the: a. fundus b. corpus c. isthmus d. cornu

d. cornu

The term that indicates the presence of two separate amniotic sacs is: a. dichorionic b. bichorionic c. monoamniotic d. diamniotic

d. diamniotic

Typically, the first sonographic manifestation of twin-twin transfusion syndrome is: a. oligohydramnios b. polyhydramnios c. dichorionic twinning d. discordant fetal growth

d. discordant fetal growth

The most superior and widest portion of the uterus is the: a. corpus b. isthmus c. cervix d. fundus

d. fundus

The location of a fibroid within the myometrium is termed: a. submucosal b. intracavitary c. subserosal d. intramural

d. intramural

Pools of maternal blood noted within the placental substance are referred to as: a. accessory lobes b. decidual casts c. chorioangiomas d. maternal lakes

d. maternal lakes

Twins having one placenta and one amniotic sac are referred to as: a. dichorionic monochorionic b. dichorionic diamniotic c. monochorionic diamniotic d. monochorionic monoamniotic

d. monochorionic monoamniotic

Twins having two placentas and one amniotic sac are referred to as: a. monochorionic diamniotic b. monoamniotic dichorionic c. dichorionic monoamniotic d. this does not occur

d. this does not occur

dyschezia

difficult or painful defecation

hydrometrocolpos

fluid accumulation within the uterus and vagina

axial skeleton

includes the bones of cranium and spine

HCG levels in abortion

low

HCG levels in anembryonic pregnancy

low

Vaginal atresia

occlusion or imperforation of the vagina. can be congenital or acquired.

What is the primary blood supply of the ovaries

ovarian arteries

tubo ovarian abscess

pelvic abscess involving the fallopian tubes and ovaries that are often caused by PID

Pelvic kidneys can cause

pelvic symptoms

achondrogenesis

rare, lethal condition resulting in abnormal development of the bones and cartilage

What can lead to postmenopausal bleeding?

some ovarian tumors

Pelvic ascites is associated with

some ovarian tumors, cirrhosis, Meigs syndrome (w/ pleural effusion and benign ovarian mass), pseudomyxoma peritonei and portal HTN

The inner cavity of the fallopian tubes can be visualized and evaluated for patency using

sonohysterography or hysterosalpingograhpy

Spina bifida is aka

spinal dysraphism, meningocele, myelomeningocele, cystica.

cleft lip

the abnormal division of the lip

retroversion

the uterine body tilts back without a bend

upper genital tract

the uterus, ovaries, and fallopian tubes

The pelvis can be divided into

true pelvis (lesser pelvis) and false pelvis (major pelvis) by an imaginary line called the linea terminalis

Lactate dehydrogenase

tumor marker for ovarian dysgerminoma

fraternal twins

twins that result from the fertilization of two separate ova and have dissimilar characteristics

identical twins

twins that result in the split of a single zygote and share the same genetic structure

dizygotic

two ova are fertilized by two sperm

therapeutic amniocentesis

type of amniocentesis used to remove a large amount of amniotic fluid around a fetus suffering from polyhydramnios

Endometrial carcinoma has been linked with

unnopposed estrogen therapy nulliparity obesity chronic anovulation (stein-leventhal PCOS) estrogen producing ovarian tumors tamoxifen

Obstruction can be the result of

vaginal atresia vaginal septum imperforate hymen

mitral valve/bicuspid valve

valve between left atrium and left ventricle

Porencephaly

A condition in which a cyst most often caused by intraparenchymal hemorrhage communicates with lateral ventricle

Aneuploidy

A condition of having an abnormal number of chromosomes

Falx cerebri

A double fold of dura mater located within the midline of the brain

Hydranencephaly

A fatal condition in which the entire cerebrum is replaced by a large sac containing cerebrospinal fluid

Meckel Gruber syndrome

A fetal syndrome associated with microcephaly, occipital encephalocele, polydactyl, polycystic kidneys

Intrauterine growth restriction is defined as: *A small-for-dates fetus *A fetus that falls below below the 10th percentile for gestational age *A fetus that is immunocompromised and has decreased umbilical cord Doppler ratios for gestational age *A fetus that fall below the 5th percentile for gestational age

A fetus that falls below below the 10th percentile for gestational age

Suture

A flexible connective tissue that lies between the cranial bones

Exencephaly

A form of acrania and which the entire cerebrum is located outside of the skull

Holoprosencephaly

A group of brain abnormalities consisting of varying degrees of fusion of the lateral ventricles absence of the midline structures and Associated facial anomalies

Arnold Chiari 2 malformation

A group of cranial abnormalities associated with spina bifida

Basal ganglia

A group of nuclei within the brain that function in several ways including information processing and emotional response

Germinal Matrix

A group of thin-walled blood vessels and cells within the subependymal layer of the fetus brain responsible for brain cell migration during fetal development

Beckwith-wiedemann syndrome

A growth disorder syndrome synonymous with enlargement of several organs including skull tongue and liver

Eventration of the diaphragm is best described as: *A lack of muscle in the dome of the diaphragm *A defect in the anterior lateral wall of the diaphragm *A defect in the posterolateral wall of the diaphragm *Congenital absence of the diaphragm

A lack of muscle in the dome of the diaphragm

Sandal Gap

A large space between the first and second toe

Nuchal cystic hygroma

A mass, typically found in the neck region, that is the result of an abnormal accumulation of lymphatic fluid within the soft tissue

Quadruple screen

A maternal blood test of HCG, AFP, estriol, inhibin a; same with triple screen but with added inhibin A.

Cavum septum pellucidum

A normal midline brain structure identified in the anterior portion of the brain between the frontal horns of the lateral ventricles

Three line sign

A periovulatory endometrial sonó appearance in which the outer echogenic basal layer surrounds the more hypoechoic functional layer, while the functional layer is separated by the echogenic endometrial stripe. Late proliferative

Alpha-fetoprotein

A protein produced by the fetal yolk Sac, fetal gastrointestinal tract, and the fetal liver; may also be produced by some malignant tumors

Pregnancy-associated plasma protein a

A protein that is produced by the placenta

Hysterosalpingography

A radiographic procedure that uses a Dye instilled into the endometrial cavity and the Fallopian tubes to evaluate for internal abnormalities

Supine hypotensive syndrome

A reduction in blood return to the maternal heart caused by the gravity uterus compressing the maternal IVC

Eclampsia

A sequela of preeclampsia in which uncontrollable maternal hypertension and proteinuria lead to maternal convulsions and possibly fetal and maternal death

Hypoxia

A shortage of oxygen or decreased oxygen in the blood

Daughter cyst

A small cyst within a large cyst

Papillary projections

A small protrusion of tissue

Dangling choroid sign

A sonographic sign associated with hydrocephalus when the choroid plexus is noted hanging freely within the dilated lateral ventricle. also a sign of ventriculomegaly.

Asherman syndrome

A syndrome characterized by endometrial adhesions that typically occur as a result of scar formation after some types of uterine surgery

Corpus callosum

A thick band of white matter that provides communication between right and left halves of the brain

Invasive mole

A type of gestational trophoblastic disease in which a molar pregnancy invades into the myometrium and may also invade through the uterine wall and into the peritoneum

Folate

A vitamin that has been shown to significantly reduce the likelihood of neural tube defects AKA folic acid

The measurement that should be carefully scrutinized in cases of IUGR is the: A. Abdominal circumference B. Femur length C. Biparietal diameter D. Head circumference

A. Abdominal circumference

The right ovarian artery branches off of the: A. Aorta B. Right renal artery C. Uterine artery D. Internal iliac artery

A. Aorta

The pelvic ligament that extends from the lateral aspect of the uterus to the side walls of the pelvis is the: A. Broad ligament B. Ovarian ligament C. Piriformis ligament D. Round ligament

A. Broad ligament

The ligament that houses the vasculature of the uterus is the: A. Cardinal ligament B. Ovarian ligament C. Broad ligament D. Suspensory ligament of the ovary

A. Cardinal ligament

An increase in the number of endometrial cells is termed: A. Endometrial hyperplasia B. Endometrial atrophy C. Endometrial carcinoma D. Polyps

A. Endometrial hyperplasia

The dominant follicle prior to ovulation is termed the: A. Graafian follicle B. corpus albicans C. corpus luteum D. medulla

A. Graafian follicle

The paired muscles that are located lateral to the uterus and anterior to the iliac crest are the: A. Iliopsoas muscles B. Rectus abdominis muscles C. Obturator interni muscles D. Piriformis muscles

A. Iliopsoas muscles

The innominate bones of the pelvis consist of: A. Ischium, ilium, and pubic bones B. Ilium, sacrum, and coccyx C. Sacrum, coccyx, and pubic bones D. Sacrum, ischium, and ilium

A. Ischium, ilium, and pubic bones

The pelvic muscle group that is located between the coccyx and the pubis is the: A. Levator ani muscles B. Rectus abdominis muscles C. Obturator internus muscles D. Piriformis muscle

A. Levator ani muscles

When the placental edge extends into the lower uterine segment but ends more than 2cm away from the internal os, it is referred to as: A. Low-lying previa B. Marginal previa C. Partial previa D. Total previa

A. Low-lying previa

The two hormones produced by the anterior pituitary gland that impact the menstrual cycle are: A. Lutenizing hormone follicle-stimulating hormone B. Lutenizing hormone and estrogen C. Progesterone and estrogen D. Follicle-stimulating hormone and progesterone

A. Lutenizing hormone follicle-stimulating hormone

Mothers with pregestational diabetes, as opposed to gestational diabetes, have an increased risk of a fetus with: a. neural tube defects b. proteinuria c. TORCH d. DES

A. Neural tube defects

The second phase of the endometrial cycle is the: A. Secretory phase B. Follicular phase C. Luteal phase D. Proliferative

A. Secretory phase

Placenta accreta denotes: A. The abnormal attachment of the placenta to the myometrium B. The premature separation of the placenta from the uterine wall C. The invasion of the placenta into the mayometrium D. The condition of having the fetal vessels rest over the internal os

A. The abnormal attachment of the placenta to the myometrium

Ovarian hyperstimulation syndrome causes multiple large follicles to develop on the ovary termed: A. Theca lutein cysts B. Chocolate cysts C. Corpus luteum cyst D. Dermoid cyst

A. Theca lutein cysts

Which of the following would be described as functional cysts that are found in the presence of elevated levels of hCG: A. Theca lutein cysts B. Chocolate cysts C. Corpus lutein cysts D. Endometrial cysts

A. Theca lutein cysts

The urinary bladder, uterus, and ovaries are located within the: A. True pelvis B. False pelvis

A. True pelvis

Doppler sonography reveals vascular structures coursing over the internal os of the cervix. This finding is indicative of: A. Vasa previa B. Placenta previa C. Placenta increta D. Abruptio placentae

A. Vasa previa

The cystic mass commonly noted with a pregnancy is the: A. corpus luteum B. dermoid cyst C. dysgerminoma D. serous cystadenoma

A. corpus luteum

The most common benign ovarian tumor is the: A. cystic teratoma B. mucinous cystadenoma C. fibroma D. sertoli-leydig cell tumor

A. cystic teratoma or Dermoid

The ovarian tumor associated with an elevated serum lactate dehydrogenase is the: A. dysgerminoma B. sertoli-leydig cell tumor C. androblastoma D. mucinous cystadenocarcinoma

A. dysgerminoma

The fingerlike extensions of the fallopian tube are called: A. fimbria B. infundibulum C. cilia D. ampulla

A. fimbria

Cancer of the fallopian tube is rare and typically in the form of what?

Adenocarcinoma

Endometrial carcinoma is most often in the form of

Adenocarcinoma

AUB abnormal uterine bleeding

Adenomyosis

Menometrorrhagia GYN diagnosis

Adenomyosis Endometrial Polyp PID Perforated intrauterine device

Enlarged uterus GYN diagnosis

Adenomyosis Endometrial carcinoma Leiomyoma (fibroid) Leiomyosarcoma

The most server form of holoprosencephaly is: A. lobar B. alobar C. semilobar D. lobular

Alobar

The protein that is produced by the yolk sac, fetal gastrointestinal tract, and the fetal liver is: A. AFP B. HCG C. Pregnancy-associated plasma protein A D. Inhibin A

Alpha-Fetoprotein

All of the following are produced by the placenta except: A. AFP B. HCG C. Pregnancy associated plasma protein A D. Inhibin A

Alpha-fetoprotein

Which protein is not produces by the developing placenta ? A. Alpha-fetoprotein B. Human chorionic gonadotropin C. Estriol D. Pregnancy-associated plasma protein A

Alpha-fetoprotein

Gestational trophoblastic disease (Molar pregnancy)

Also referred to as molar pregnancy; is associated with an abnormal proliferation of the trophoblastic cells, enlargement of the placenta, and elevated levels of human chorionic gonadotropin

The birth defect in which the sex of the fetus can't be determined defines: Renal agenesis Ovarian dysgenesis Clitorimegaly Ambiguous genitalia

Ambiguous genitalia

A group of abnormalities associated w/ the entrapment of fetal parts and fetal amputations is: Cystic hygroma Edwards syndrome Ethmocephaly Amniotic band syndrome

Amniotic band syndrome

Limb-body wall complex is also associated with

Amniotic band syndrome

The disorder associated w/ fetal amputations is: Achondroplasia Osteogenesis imperfecta Thanatophoric dysplasia Amniotic band syndrome

Amniotic band syndrome

Hydrops

An abnormal accumulation of fluid in at least two fetal body cavities

A cystic hygroma is the result of: *Alcohol consumption in the 1st trimester *An abnormal development of the roof of the 4th ventricle *Occlusion of the internal carotid arteries *An abnormal accumulation of lymphatic fluid w/in the soft tissue

An abnormal accumulation of lymphatic fluid w/in the soft tissue

Anembryonic gestation

An abnormal pregnancy in which there is no evidence of a fetal Pole or yolk Sac within the gestational Sac A K A blighted ovum

Subarachnoid space

An area located between the arachnoid membrane and Pia Mater

Doliochocephaly

An elongated narrow head shape AKA scaphocephaly

Prune belly syndrome is caused by: *An enlarged bladder *Unilateral renal agenesis *Bilateral renal agenesis *Hyopspadias

An enlarged bladder

Mega cisterna magna

An enlargement of the cisterna magna as defined by depth of more than 10 millimeter

Estriol

An estrogenic hormone produced by the placenta

Sonohysterography

An ultrasound procedure that uses saline instillation into the endometrial cavity and Fallopian tubes to evaluate for internal abnormalities aka saline infusion sonography

Which of the following would not typically produce an elevation in human chorionic gonadotropin? A. Down syndrome B. Anembryonic pregnancy C. Triploidy D. Molar pregnancy

Anembryonic pregnancy

Pathologies with High MSAFP

Anencephaly Cephalocele Gastroschisis/omphalocele Patau syndrome (trisomy 13) mildly increased Spina Bifida (meningocele or myelomeningocele) closed not open

What are the most common neural tube defects?

Anencephaly and spina bifida

Acrania ( anencephaly and excencephaly)

Anencephaly: considered when there are NO cerebral hemispheres Excencephaly: denotes a normal amount of cerebral tissue

The absence of the eyes is termed: A. Agyria B. Epignathus C. Hypotelorism D. Anophthalmia

Anophthalmia

The congenital maldevelopment of the rectum and absence of anal opening is termed: Jejunal atresia Intussusception Anorectal atresia Duodenal atresia

Anorectal atresia

Which of the following is considered to be the most common type of colonic atresia? Duodenal atresia Jejunal atresia Anorectal atresia Intussusception

Anorectal atresia

Which of the following would be least likely to be associated w/ an elevated maternal serum alpha-fetoprotein? Pentalogy of Cantrell Anorectal atresia Gastroschisis Omphalocele

Anorectal atresia

Climacteric

Another name for menopause

Periovulatory phase

Another name for the late proliferative phase of the endometrial cycle, which occurs around the time of ovulation

Fontanelle closure times

Anterior (Frontal) - by 18 months Posterior (Occipital) - by 5 months Antero/posterolateral (sphenoidal/mastoid) (2 each) - by 2 years

Types of fontanelles and locations

Anterior (frontal) - bordered by the frontal and parietal bones Posterior (occipital) - bordered by the occipital bones Anterolateral (sphenoidal) (2) - bordered by the frontal, parietal, and sphenoid bones Posterolateral (mastoid) (2) - bordered by the mastoid and occipital bones

The most common cause of hypertelorism is: *Dandy-Walker malformation *Anencephaly *Anterior cephalocele *Holoprosencephaly

Anterior cephalocele

The fourth ventricle is located: A. posterior to the cavum septum pellucidum B. between the frontal horns of the lateral ventricles C. anterior to the cerebellar vermis D. medial to the third ventricle

Anterior to the cerebellar vermis

The normal position of the UT is called

Anteversion or Anteflexion

Vasculature of the uterus

Aorta branches of ovarian arteries/Internal Iliac A Uterine arteries (wall of uterus) Arcuate arteries Radial arteries Straight /Spiral arteries

What supplies the blood to the female genitalia

Aorta then CIA then the internal iliac arteries

The third ventricle communicates with the fourth ventricle at the: A. foramen of magendie B. foramen of luschka C. foramen of monro D. aqueduct of sylvius

Aqueduct of Sylvius

The most common cause of hydrocephalus in utero is: A. cerebral hemorrhage B. holoprosencephaly C. brain tumors D. Aqueductal stenosis

Aqueductal stenosis

Cisterna magna meas that is <2mm is consistent with

Arnold Chiari II malformation

Spina bifida causes malformations called

Arnold chiari II malformation

The group of fetal head and brain abnormalities that often coexists w/ spina bifida is referred to as: Dandy-Walker malformation Budd-Chiari syndrome Arnold-Chiari II malformation Amniotic band syndrome

Arnold-Chiari II malformation

Ectopic endometrial tissue undergoes physiologic changes?

As a result of stimulation of the hormones of the menstrual cycle

Abdominal distension GYN diagnosis

Ascites Leiomyoma (Fibroid) Ovarian hyperstimulation syndrome Ovarian malignancy

When can ovarian torsion happen?

At any time even in a fetus.

The "keyhole" sign would be seen in all of the following situations except: *Urethal atresia *Prune belly syndrome *Autosomal dominant polycystic renal disease *Posterior urethral valves

Autosomal dominant polycystic renal disease

Which of the following is associated w/ enlarged echogenic kidneys and microscopic renal cysts? *Multicystic dysplastic kidney disease *Obstructive cystic dysplasia *Hydronephrotic syndrome *Autosomal recessive polycystic kidney disease

Autosomal recessive polycystic kidney disease

The placenta is considered too thick when it measures: a. >4mm b. >4cm c. >8mm d. >3.5cm

B. >4cm

A change in menstrual bleeding associated with lesions within the uterus relates to: A. Dysfunctional uterine bleeding B. Abnormal uterine bleeding C. Pelvic inflammatory disease D. Fibroids

B. Abnormal uterine bleeding

Ectopic endometrial tissue within the uterus that leads to abnormal uterine bleeding is termed: A. Endometriosis B. Adenomyosis C. Fibroids D. Endometrial hyperplasia

B. Adenomyosis

The temporary endocrine gland that results from the rupture of the graafian follicle is the: A. Corpus albicans B. Corpus luteum C. Cumulus oophorus D. Trophoblastic cells

B. Corpus luteum

Normally, the S/D ratio: A. Increases with advancing gestation B. Decreases with advancing gestation C. Reverses occasionally during a normal pregnancy D. Has an absent diastolic component

B. Decreases with advancing gestation

Endocrine abnormalities that cause abnormal vaginal bleeding are related to: A. Hirsutism B. Dysfunctional uterine bleeding C. Fibroids D. Pelvic inflammatory disease

B. Dysfunctional uterine bleeding

Evidence of polhydramnios should warrant a careful investigation of the fetal: A. Genitourinary system B. GI system C. Extremeties D. Cerebrovascular system

B. GI system

The malignant ovarian tumor with GI origin is the: A. Brenner tumor B. Krukenberg Tumor C. yolk sac tumor D. granulosa tumor

B. Krukenberg Tumor

The periovulatory phase may also be referred to as the: A. Early secretory phase B. Late proliferative phase C. Late secretory phase D. Early proliferative

B. Late proliferative phase

Weakening of these muscles may lead to prolapse of the pelvic organs: A. Rectus abdominis and obturator internis muscles B. Levator ani and coccygeus muscles C. Obturator internus and levator ani muscles D. Piriformis and iliopsoas muscles

B. Levator ani and coccygeus muscles

The second phase of the ovarian cycle is called the: A. Follicular phase B. Luteal phase C. Secretory phase D. Proliferative phase

B. Luteal phase

The ovary is supplied blood by the: A. Ovarian artery B. Ovarian artery and uterine artery C. Uterine artery D. Arcuate artery

B. Ovarian artery and uterine artery

The pelvic ligament that provides support to the ovary and extends from the lateral surface of the ovary to the uterus is the: A. Cardinal ligament B. Ovarian ligament C.Broad ligament D. Suspensory ligament of the ovary

B. Ovarian ligament

The pelvic ligament that provides support to the ovary and extends from the ovary to the lateral surface of the uterus is the: A. Cardinal ligament B. Ovarian ligament C. Broad ligament D. Suspensory ligament

B. Ovarian ligament

The corpus luteum primarily releases: A. Estrogen B. Progesterone C. Luteninizing hormone D. Follicle-Stimulating hormone

B. Progesterone

The presence of pus within the uterus defines: A. Pyosalpinx B. Pyometra C. Pyocolpos D. Pyomyoma

B. Pyometra

Sonographic findings of the endometrium in a patient with a history of pelvic inflammatory disease, fever, and elevated white blood cell count would include all of the following except: A. Ring-down artifact posterior to the endometrium B. Thin, hyperechoic endometrium C. Endometrial fluid D. Thickened, irregular endometrium

B. Thin, hyperechoic endometrium

A patient presents to the sonography department with a fever, chills, and vaginal discharge. Sonograhically, what findings would you most likely not encounter? A. Cul-de-sac fluid B. Uterine adhesions C. Dilated uterine tubes D. Ill-defined uterine border

B. Uterine adhesions

Fluid noted anterior to the uterus would most likely be located within the: A. Pouch of Douglas B. Vesicouterine pouch C. Space of Retzius D. Rectouterine pouch

B. Vesicouterine pouch

Normal ovarian flow is said to be: A. low resistant during menstruation and high resistant during the proliferative phase B. high resistant during menstruation and low resistant at the time of ovulation C. Low resistant D. High resistant

B. high resistant during menstruation and low resistant at the time of ovulation

The inner layer of the wall of the fallopian tube is the: A. muscular layer B. mucosal layer C. myometrial layer D. serosal layer

B. muscosal layer

All of the following adnexal masses may appear sonographically similar to a uterine leiomyoma except: A. thecoma B. paraovarian cyst C. fibroma D. granulosa cell tumor

B. paraovarian cyst

The ovarian cyst associated with gestational trophoblastic disease is the: A. corpus luteum cyst B. theca lutein cyst C. dermoid cyst D. paraovarian cyst

B. theca lutein cyst

Hysterosalpingography utilizes what substance for the visualization of the uterine cavity and fallopian tubes: A. saline B. x-ray dye C. water D. betadine

B. x-ray dye

The growth disorder syndrome synonymous w/ organ, skull, and tongue enlargement is: *Klinefelter syndrome *Apert syndrome *Meckel-Gruber syndrome *Beckwith-Wiedermann syndrome

Beckwith-Wiedermann syndrome

Arachnoid cyst

Benign cyst within the brain that do not communicate with the ventricular system

Cystic teratoma (Dermoid)

Benign ovarian mass that is composed of the three germ cell layers

Endometrioma

Benign, blood containing tumor that forms from the implementation of ectopic endometrial tissue, tumor associated with endometriosis

The third ventricle is located: A. Anterior to the thalamus B. anterior to the cerebellar vermis C. between the two lobes of the thalamus D. superior to the corpus callosum

Between the two lobes of the thalamus

Which of the following would be most likely associated w/ oligohydramnios? Duodenal atresia Hepatomegaly Bilateral renal agenesis Physiologic bowel herniation

Bilateral renal agenesis

The measurement obtained b/n the lateral walls of the orbits is referred to as the: Interocular diameter Binocular diameter Ocular diameter Biparietal diameter

Binocular diameter

Is adenomyosis focal or diffuse?

Both

Both Tubo Ovarian Complex vs Abcess Sonographic Findings

Both 1. Thickened, Irregular Endometrium 2. Pyosalpinx / Hydrosalpinx 3. Cul de sac fluid 4. Multicystic and solid complex adnexal masses Complex - Ovaries and tubes recognized as distinct structures but the ovaries will not be seperated by pushing them with a vaginal probe. Abscess - Complete loss of borders of all adnexal structures and development of conglomerated adnexal masses

Rounded head shape is referred to as: A. Dolichocephaly B. Brachycephaly C. Cebocephaly D. Craniosynostosis

Brachycephaly

Brenner tumors Fibroma Thecoma

Brennor - Small benign ovarian tumors Fibroma - An ovarian sex cord stromal tumor found in middle aged women Thecoma - Benign ovarian sex cord stromal tumor that produces estrogen in older woman

An 84-year-old patient presents to the ultrasound department with sudden onset of vaginal bleeding. Her endometrium should not exceed: A. 6 mm B. 8 mm C. 5 mm D. 3 mm

C. 5 mm

Increased S/D ratio is associated with all of the following except: A. IUGR B. Placental insufficiency C. Allantoic cysts D. Perinatal mortality

C. Allantoic cysts

Doppler assessment of the middle cerebral artery: A. Helps to determine if fetal anorexia is occurring B. Is valuable in diagnosing the extent of ventriculomegaly C. Can evaluate the fetus for hypoxia D. Is important to determine if TORCH complications are present

C. Can evaluate the fetus for hypoxia

After the corpus luteum has regressed, which structure remains? A. Theca luteal cysts B. Corpus luteum of pregnancy C. Corpus albicans D. Cumulus oopherus

C. Corpus albicans

The structure noted within the graafian follicle containing the developing ovum is the: A. Corpus luteum B. Corpus albicans C. Cumulus oophorus D. Theca internal cells

C. Cumulus oophorus

Asherman syndrome is associated with: A. Uterine leiomyoma B. Endometrial polyp C. Endometrial adhesions D. Ovarian fibroma

C. Endometrial adhesions

Which hormone released by the ovary during the proliferative phase stimulates endometrial thickening? A. Follicular-stimulating hormone B. Lutenizing hormone C. Estrogen D. Progesterone

C. Estrogen

All of the following are associated with a thin placenta except: A. Preeclampsia B. IUGR C. Fetal hydrops D. Long-standing diabetes

C. Fetal hydrops

Meigs Syndrome is associated with ovarian tumor? A. ovarian cystadenoma B. cystic teratoma C. Fibroma D. Yolk sac tumor

C. Fibroma

The hormone of the pituitary gland that stimulates follicular development of the ovary is: A. Luteinizing hormone B. Estrogen C. Follicle-stimulating hormone D. Gonadotropin-releasing hormone

C. Follicle-stimulating hormone

The hormone produced by the hypothalamus that controls the release of the hormones for menstruation by the anterior pituitary gland is: A. Follicular-stimulating hormone B. Estrogen C. Gonadtotropin-releasing hormone D. Lutenizing

C. Gonadtotropin-releasing hormone

The right ovarian vein drains directly into the: A. Right renal vein B. Aorta C. Inferior vena cava D. Common iliac vein

C. Inferior vena cava

The uterine artery branches off of the: A. Abdominal aorta B. Uterine plexus C. Internal iliac artery D. External iliac artery

C. Internal iliac artery

Fetal TORCH is frequently associated with: A. Maternal hypertension B. Twin-twin transfusion syndrome C. Intracranial calcifications D. Renal cystic disease

C. Intracranial calcifications

The true pelvis is delineated from the false pelvis by the: A. Space of Retzius B. Adnexa C. Linea terminalis D. Iliac crest

C. Linea terminalis

The hormone that is responsible for ovulation is: A. Gonadotropin-releasing hormone B. Aldosterone C. Luteinizing hormone D. Progesterone

C. Luteinizing hormone

Insertion of the umbilical cord at the edge of the placenta is referred to as: A. Velamentous cord insertion B. Partial cord insertion C. Marginal cord insertion D. Unchallenged cord insertion

C. Marginal cord insertion

The muscle located lateral to the ovaries is the: A. Iliopsoas muscle B. Rectus abdominis muscle C. Obturator internus muscle D. Piriformis muscle

C. Obturator internus muscle

One of the most common causes of painless vaginal bleeding in the second and third trimester is: A. Spontaneous abortion B. Abruptio placentae C. Placenta previa D. Placenta accreta

C. Placenta previa

All of the following are associated with a thick placenta except: A. Fetal infections B. Rh isoimmunization C. Placental insufficiency D. Multiple gestations

C. Placental insufficiency

Another name for the rectouterine pouch is the: A. Space of Retzius B. Pouch of Retzius C. Pouch of Douglas D. Anterior cul-de-sac

C. Pouch of Douglas

A velamentous cord insertion is associated with which of the following: A. Placenta increta B. Placental abruption C. Vasa previa D. Circumvallate placenta

C. Vasa previa

Hairlike projections within the fallopian tube are called: A. interstitia B. fimbria C. cilia D. peristalsis

C. cilia

After the graafian follicle ruptures, the remaining structure is termed the: A. Graafian remnant B. Corpus albicans C. corpus Luteum D. theca lutein cyst

C. corpus luteum

Which of the following is the correct fomula for calculating ovarian volume? A. length x width x height x 0.6243 B. length x width x height x 0.3899 C. length x width x height x 0.5233 D. ovarian volume cannot be calculated

C. length x width x height x 0.5233

Which of the following is an estrogen-producing ovarian tumor? A. Brenner tumor B. fibroma C. thecoma D. Endometrioma

C. thecoma

The foramen of magendie allows

CSF to pass from the 4th vent to the cisterna magna and subarachnoid space.

The foramen of luschka allow

CSF to travel from the 4th ventricle to the subarachnoid space around the brain

An absent sacrum and coccyx is referred to as: Sirenomelia Caudal regression syndrome Achondroplasia Radial ray defect

Caudal regression syndrome

The condition associated w/ the absence of the sacrum and coccyx: *Limb-body wall complex *Caudal regression syndrome *Thanatophoric dwarfism *Heterozygous achondroplasia

Caudal regression syndrome

The anechoic midline brain structure located between the frontal horns of the lateral ventricles is the: A. cavum septum pellucidum B. cavum vergae C. corpus callosum D. fourth ventricle

Cavum septum pellucidum

Close-set eyes and a nose w/ a single nostril is termed: Cebocephaly Cyclopia Ethmocephaly Epignathus

Cebocephaly

Theca internal cells

Cells of the follicle that produce estrogen

Of the following forms of fetal presentation, with is the most common: A. Cephalic B. Complete breech C. Frank breech D. Transverse

Cephalic

Presentation types

Cephalic Breech (complete, incomplete, frank) Complete Breech - fetal legs flexed at hips and flexion of knees Frank Breech - fetal butt is closest to the cx Footling or incomplete breech - extension of at least one leg toward the cx

The most common causes of PID is?

Chlamydia and Gonorrhea

Placental tissue is obtained with what procedure? A. Amneocentisis B. Cordocentesis C. Chorionic villi sampling D. Trophoblastic resection technique

Chorionic villi sampling

The earliest fetal karyotyping technique that can be performed is: A. Amniocentesis B. Cordocentisis C. Chorionic villi sampling D. Percutaneous umbilical cord sampling

Chorionic villi sampling

An abnormal division in the lip is referred to as: Micrognathia Cleft lip Anophthalmia Cebocephaly

Cleft lip

Drug that is used to stimulate the pituitary gland to secrete increased amounts of FSH

Clomid

Cebocephaly

Close-set eyes hypothyroidism and a nose with a single nostril

Talipes equinovarus is associated w/: Clubfoot Syndactyly Rhizomelia Rockerbottom feet

Clubfoot

The narrowing of the aortic arch is indicative of: *Tetralogy of Fallot *Coarctation of the aorta *Ebstein anomaly *Hypoplastic right heart syndrome

Coarction of the aorta

Heterotopic pregnancy

Coexisting ectopic and intrauterine pregnancies

The condition in which the frontal horns are small and the occipital horns are enlarged is referred to as: A. Ethmocephaly B. hydrocephalus C. colpocephaly D. encephalitis

Colpocephaly

Cordocentesis Amniocentesis Chorionic villi sampling

Cordocentesis - fetal blood for chromosomal analysis Amniocentesis - amniotic fluid is extracted for genetic testing or removed when there is an accumulation of an excessive amount of fluid around the fetus Chorionic villi sampling - placental tissue for chromosomal analysis

Fetal head suture types

Coronal - Between the frontal and two parietal bones Sagittal - Between the two parietal bones Lambdoidal suture - Between the parietal bones and occipital bone Squamosal - Between the parietal bones and temporal bones Metopic - Located within the frontal bone along the midline of the forhead

What is the most common pelvic mass associated with pregnancy?

Corpus Luteum cyst

With endometrial atrophy, the endometrial thickness should not exceed: A. 6 mm B. 8 mm C. 5 mm D. 3 mm

D. 3 mm

Follicular-stimulating hormone is produced by the: A. Ovary B. Endometrium C. Hypothalamus D. Anterior pituitary gland

D. Anterior pituitary gland

Lutenizing hormone is produced by the: A. Ovary B. Endometrium C. Hypothalamus D. Anterior pituitary gland

D. Anterior pituitary gland

The peripheral arteries of the uterus are the: A. Radial arteries B. Spiral arteries C. Straight arteries D. Arcuate arteries

D. Arcuate arteries

Assisted reproductive therapy can result in all of the following except: A. Heterectopic pregnancy B. Multiple gestations C. Ovarian-hyperstimulating syndrome D. Asherman syndrome

D. Asherman syndrome

The development of adhesions within the uterine cavity is termed: A. Fitz-Hugh-Curtis syndrome B. Dandy-Walker syndrome C. Stein-Levanthal syndrome D. Asherman syndrome

D. Asherman syndrome

IUGR is evident when the EFW is: A. Above the 90th percentile B. Below the 90the percentile C. Above the 10th percentile D. Below the 10th percentile

D. Below the 10th percentile

The best description for endometrial polyps is: A. Malignant nodules that cause bleeding B. Benign lesions associated with cervical stenosis C. Malignant nodules that are associated with endometrial atrophy D. Benign nodules of hyper plastic endometrial tissue

D. Benign nodules of hyper plastic endometrial tissue

The space of Retzius is located: A. Between the uterus and bladder B. Between the bladder and ilium C. Along the lateral aspect of the uterus D. Between the bladder and pubic bone

D. Between the bladder and pubic bone

All of the following are associated with polyhydramnios except: A. Omphalacele B. Gastroschisis C. Esophageal atresia D. Bilateral multicystic dysplastic kidney disease

D. Bilateral multicystic dysplastic kidney disease

Fetal contribution to the placenta is the: a. chorionic vera b. decidua vera c. decidua basalis d. chorion frondosum

D. Chorion frondosum

All of the following are association with oligohydramnios except: A. Bilateral renal agenesis B. Infantile polycystic kidney disease C. Premature rupture of membranes D. Duodenal atresia

D. Duodenal atresia

A 67-year-old patient on hormone replacement therapy presents to the ultrasound department with abnormal uterine bleeding. Sonographically, the endometrium measures 9 mm and contains small cystic areas. The most likely cause of her bleeding is: A. Endometrial carcinoma B. Asherman syndrome C. Endometrial polyps D. Endometrial hyperplasia

D. Endometrial hyperplasia

The first phase of the ovarian cycle is the: A. Luteal phase B. Secretory phase C. Proliferative phase D. Follicular phase

D. Follicular phase

All of the following are clinical features of placnental abruption except: A. Vaginal bleeding B. Uterine tenderness C. Abdominal pain D. Funneling of the cervix

D. Funneling of the cervix

The dominant follicle prior to ovulation is termed the: A. Ovarian hyper follicle B. Corpus luteum C. Corpus albicans D. Graafian follicle

D. Graafian follicle

The corpus luteum is maintained during pregnancy by which hormone? A. Follicle-stimulating hormone B. Luteinizing hormone C. Progesterone D. Human chorionic gonadotropin

D. Human chorionic gonadotropin

The hormone produced by the trophoblastic cells of the early placenta: A. Gonadotropin-releasing hormone B. Follicle-stimulating hormone C. Luteinizing hormone D. Human chorionic gonadotropin

D. Human chorionic gonadotropin

Pelvic bones, when visualized on sonography, will appear: A. Anechoic B. Hypoechoic C. Dark D. Hyperechoic

D. Hyperechoic

The left ovarian vein drains directly into the: A. Right renal vein B. Inferior vena cava C. Aorta D. Left renal vein

D. Left renal vein

Mothers with gestational diabetes run the risk of having fetuses that are considered: A. Nutritionally deficient B. Acromegalic C. Microsomes D. Macrosomic

D. Macroscomic

The first menstrual cycle is termed: A. Amenorrhea B. Metrorrhagia C. Mittelschmerz D. Menarche

D. Menarche

Sonographic findings of ovarian hyper stimulation syndrome include all of the following except: A. Cystic enlargement of the ovaries B. Ascites C. Pleural effusion D. Oliguria

D. Oliguria

During a pelvic sonogram you visualize a small cyst located adjacent to the ovary. What is the most likely etiology of this cyst? A. Dermoid cyst B. ovarian cystadenoma C. endometrioma D. Parovarian Cyst

D. Paraovarian cyst

The muscles that may be confused with the ovaries on a pelvic sonogram include the: A. Rectus abdominis and obturator internus muscle B. Levator ani and coccygeus muscle C. Obterator internus and levator ani muscles D. Piriformis and iliopsoas muscle

D. Piriformis and iliopsoas muscle

The bilateral muscles that are located posterior to and extend from the sacrum to the femoral greater trochanter are the: A. Levator ani muscles B. Rectus abdominis muscles C. Obturator internus muscles D. Piriformis muscles

D. Piriformis muscles

Which of the following hormones is released by the ovary during the second half of the menstrual cycle? A. Lutenizing hormone B. Follicle-stimulating hormone C. Human chorionic gonadotropin D. Progesterone

D. Progesterone

The first phase of the endometrial cycle is the: A. Secretory phase B. Follicular phase C. Luteal phase D. Proliferative phase

D. Proliferative phase

The normal umbilical cord has: A. One vein and one artery B. Two veins and two arteries C. Two veins and one artery D. Two arteries and one vein

D. Two arteries and one vein

The sonographic appearance of a 59-year old woman on hormone replacement therapy is: A. Hypoechoic and thickened B. Hyperechoic and thickened C. Cystic areas within a thickened endometrium D. Variable depending upon the menstrual cycle

D. Variable depending upon the menstrual cycle

The anterior cul-de-sac is also referred to as the: A. Space of Retzius B. Rectouterine pouch C. Pouch of Douglas D. Vesicouterine pouch

D. Vesicouterine pouch

The segment of the fallopian tube where fertilization typically occurs is the: A. cornu B. fimbria C. interstitial D. ampulla

D. ampulla

Which of the following is a tumor of ectopic endometrial tissue? A. Brenner tumor B. cystic teratoma C. yolk sac tumor D. endometrioma

D. endometrioma

The most distal part of the fallopian tube is the: A. cornu B. fimbria C. interstital D. infundibulum

D. infundibulum

The short and narrow segment of the fallopian tube distal to the interstitial segment is the: A. ampulla B. fimbria C. infundibulum D. isthmus

D. isthmus

The malignant ovarian mass that is associated with pseudomyxoma peritonei is the: A. dysgerminoma B. sertoli-Leydig cell tumor C. serous cystadenocarcinoma D. mucinous cystadenocarcinoma

D. mucinous cystadenocarcinoma

The ovarian cyst associated with distinctively elevated levels of hCG is the: A. corpus luteum cyst B. parovarian cyst C. Dermoid Cyst D. theca lutein cyst

D. theca lutein cyst

A patient with an ovarian mass presents with an elevated MSAFP. Which of the following would be most likely? A. ovarian fibroma B. ovarian thecoma C. cystic teratoma D. yolk sac tumor

D. yolk sac tumor

Agenesis or hypoplasia of the cerebellar vermis results in what cerebral malformation? A. Arnold-chiari II malformation B. schizencephaly C. mega cisterna magna D. dandy-walker malformation

Dandy-Walker Malformation

Elevation of the tentorium and an enlarged cisterna magna that communicates with a dilated fourth ventricle is consistent with: A. Arnold-chiari II malformation B. schizencephaly C. mega cisterna magna D. dandy-walker malformation

Dandy-Walker Malformation

All of the following are clinical or sonographic findings consistent w/ limb-body wall complex except: *Ventral wall defects *Decreased MSAFP *Marked scoliosis *Shortened umbilical cord

Decreased MSAFP

The maternal serum screening ofa A mother with trisomy 18 will reveal : A. Decreased human chorionic gonadotropin, elevated Alpha-fetoprotein, and normal estriol B. Increased human chorionic gonadotropin, Alpha-fetoprotein, and estriol C. Increased Alpha-fetoprotein, increased human chorionicgonadotropin, and decreasedestriol D. Decreased human chorionicgonadotropin, Alpha-fetoprotein, and estriol

Decreased human chorionic gonadotropin, Alpha-fetoprotein, and estriol

Atresia ovarian follicle

Degeneration of a follicle

The most common cause of cardiac malposition is: Diaphragmatic hernia Omphalocele Gastroschisis Pulmonary hypoplasia

Diaphragmatic hernia

Nuchal thickening is most commonly associated w/: Patau syndrome Hydranencephaly Down syndrome Cebocephaly

Down syndrome

Which of the following is also referred to as Trisomy 21? Edwards syndrome Patau syndrome Meckel-Gruber syndrome Down syndrome

Down syndrome

Which of the following would most likely not be associated with an elevation in maternal serum alpha-fetoprotein? A. Anencephaly B. Down Syndrome C. Meningocele D. Myelomeningocele

Down syndrome

Widened pelvic angles and duododnal atresia are most consistent with the sonographic markers for: A. Triploidy B. Patau syndrome C. Down syndrome D.Edwards syndrome

Down syndrome

Blood is shunted into the IVC from the umbilical vein by what structure? Ductus venosus Ductus arteriosus Foramen ovale Foramen of Luschka

Ductus venosus

Congenital maldevelopment of the proximal portion of the small intestine is termed: *VACTERL syndrome *Esophageal atresia *Duodenal atresia *Jejunal atresia

Duodenal atresia

The "double bubble" sign is indicative of: Esophageal atresia Duodenal atresia Hydrocephalus Oligohydramnios

Duodenal atresia

Which of the following would be most likely associated w/ an excessive amount of amniotic fluid? Duodenal atresia Hepatomegaly Bilateral renal agenesis Physiologic bowel herniation

Duodenal atresia

Which of the following is the most common renal anomaly? A. Horseshoe kidneys B. Pelvic kidneys C. Renal agenesis D. Duplex collecting system

Duplex collecting system

Malignant Tumors of the ovaries

Dysgerminoma Yolk sac tumor serous/mucinous Cystadenocarcinoma Krukenburg Tumor Sertoli Leydig

Which of the following is best defined as difficult or painful menstruation? A. Dysmenorrhea B. Dyspareunia C. Dysuria D. Menorrhagia

Dysmenorrhea

Endometrial thickness by stages

Early proliferative - 4-8mm Periovulatory - 6-10mm Secretory - 7-14mm

Typically, with anencephaly, the maternal serum alpha-fetoprotein value will be: A. Elevated C. Decreased B. This laboratory finding is not helpful D. unchanged

Elevated

Typically, with gastroschisis, the maternal serum alpha-fetoprotein value will be: A. Elevated C. Decreased B. This laboratory finding is not helpful D. unchanged

Elevated

All of the following are characteristics of spina bifida occulta except: Closed defect Elevated MSAFP Sacral dimple Hemangioma

Elevated MSAFP

Clinical findings of spina bifida aperta (cystica)

Elevated MSAFP

Neural tube

Embryologic formation that results from the fusion of the two folded ends of the neural plate

Cloacal exstrophy is associated w/ all of the following except: Omphalocele Spina bifida Encephalocele Imperforate anus

Encephalocele

Yolk Sac tumor AKA?

Endodermal sinus tumor

What is the most common malignancy in the female genial tract?

Endometrial carcinoma.

What is a common cause of abnormal vaginal bleeding?

Endometrial hyperplasia

Menorrhagia GYN diagnosis

Endometrial hyperplasia Endometriosis Leiomyoma Leiomyosarcoma

Intermenstrual bleeding GYN diagnosis

Endometrial polyp

Elevated CA-125 GYN diagnosis

Endometriosis Leiomyoma (fibroid) Ovarian carcinoma PID

Constipation or painful bowel movements GYN Diagnosis

Endometriosis (endometrioma) Leiomyoma/sarcoma Ovarian mucinous/serous cystadenocarcinoma/adenoma

The "keyhole" sign describes the sonographic findings of a/an: *Enlarged bladder and dilated urethra *Bilateral renal agenesis *Unilateral renal agenesis *Dilation of the renal pelvis and proximal ureter

Enlarged bladder and dilated urethra

All of the following are associated w/ spina bifida except: *Splaying of the laminae *Enlarged posterior fossa *Lemon sign *Hydrocephalus

Enlarged posterior fossa

An oral teratoma is referred to as: Macroglossia Epignathus Micrognathia Ethmocephaly

Epignathus

Bladder exstrophy describes: *Absence of the cloaca *Protrusion of the bladder into the umbilicus *External position of the bladder *Enlargement of the bladder

External position of the bladder

Pallor

Extreme paleness of the skin

Primary amenorrhea

Failure to experience menarche before age 16

All of the following may be visualized at the correct level of the head circumference except: A. Third Ventricle B. Thalamus C. Cavum Septum Pellucidum D. Falx cerebelli

Falx cerebelli

The double fold of dura mater used to divide the cerebral hemispheres is the: A. cerebellum B. cavum septum pellucidum C. corpus callosum D. falx cerebri

Falx cerebri

What is located within the interhemispheric fissure?

Falx cerebri double fold of duramater

What treatment will also result in the development of multiple, enlarged follicular cysts?

Fertility treatment

All of the following are observed during a biophysical profile except: A. Fetal Tone B. Thoracic Movmt C. Fetal Breathing D. Fetal Circulation

Fetal circulation

A co-existing pericardial effusion and a pleural effusion is consistent w/ the diagnosis of: *Tetralogy of Fallot *Petnalogy of Cantrell *Fetal hydrops *Potter syndrome

Fetal hydrops

Non-immune hydrops

Fetal hydrops caused by congenital fetal anomalies and infections

RUQ pain GYN diagnosis

Fitz-Hugh-Curtis syndrome

Proboscis

Flesh- tongue like appendage that is typically located within the midline above the orbits in association with cyclopia and holoprosencephaly

Hemorrhage of endometriosis tissue occurs resulting in what?

Focal areas of bloody tumors - endometriomas or Chocolate cysts

What reduces the change of fetus having a neural tube defect?

Folate

What are the ovaries stimulated by?

Follicle-stimulating hormone

Ovary and endometrial phase day 1-14

Follicular (OV) Proliferative (Endo)

The most common form of diaphragmatic hernia is the: Foramen of Morgagni Foramen of Magendie Foramen of Luschka Foramen of Bochdalek

Foramen of Bochdalek

What is the normal opening in the lower middle third of the atrial septum? *Foramen of Magendie *Foramen of Monro *Foramen ovale *Ductus arteriosus

Foramen ovale

What is the fetal presentation when the fetal buttocks are closest to the cervix referred to? A. Footling Breech B. Frank Breech C. Complete breech D. Transverse

Frank breech

Pentalogy of Cantrell includes all of the following findings except: Cardiovascular malformations Diaphragmatic malformations Omphalocele Gastroschisis

Gastroschisis

Intracranial hemorrhage

General term used to denote a hemorrhage within the cranium

Endovaginal transducers are often soaked in some form of: A. Glutaraldehyde B. Ascites C. Formaldehyde D. Alcohol

Glutaraldehyde

What is a Malignant estrogen producing tumor that is considered to be the most common estrogenic tumor?

Granulosa cell tumor - most common estrogenic tumor Thecoma - produces estrogen but benign

The triple screen typically includes an analysis of: A. HCG, AFP, Estriol B. Fetal Nuchal translucency, AFP, Inhibin A C. HCG, AFP, Inhibin A D. HCG, AFP, pregnancy-associated plasma protein A

Hcg, alpha-fetoprotein, and estriol

The most accurate measurement at estimating gestational age from the list below is: A. biparietal diameter B. head circumference C. transcerebellar measurement D. lateral ventricle

Head circumference

Bilateral brain structures

Hemispheres of the cerebellum Hemispheres of the cerebrum Lobes of the thalamus Foramen of Monro (interventricular foramina) Lateral ventricles Choroid plexus (within lateral ventricles)

Intraventricular hemorrhage

Hemorrhage located within the ventricles of the brain

Atypical facial features are most commonly associated with what cerebral abnormality listed? A. dandy-walker malformation B. schizencephaly C. Lissencephaly D. holoprosencephaly

Holoprosencephaly

Facial anomalies, when discovered, should prompt the sonographer to analyze the brain closely for signs of: Holoprosencephaly Dandy-Walker malformation Schizencephaly Hydranencephaly

Holoprosencephaly

The nonexistence of corpus callosum and cavum septum pellucidum has been linked with

Holoprosencephaly Dandy-walker malformation Aqueductal stenosis T18 T8 T13

How does the normal ovary appear?

Homogeneous with a medium level too low level echogenicity, multiple follicles may be noted during neonatal and pre-pubertal ages

Patients with Polycystic Ovarian Syndrome may suffer from chronic anovulation as a result from?

Hormonal Imbalances

Follicle-stimulating hormone (FSH)

Hormone of the anterior pituitary gland that causes the development of multiple follicles on the ovaries during the first half of the menstrual cycle (follicular phase)

The sonographic finding of a fluid-filled cranium with absence of cerebral tissue is consistent with: A. Hydrocephalus B. hydranencephaly C. holoprosencephaly D. schizencephaly

Hydrancephaly

A cystic hygroma is found in all of the following condition except: Edwards syndrome Hydranencephaly Turner syndrome All of the above

Hydranencephaly

A macrocephalic headshape is associated with

Hydrocephalus hydrancephaly intracranial tumors familial inheritance beckwith-wiedemann syndrome

A group of anomalies characterized by a small or absent left ventricle is: *Turner syndrome *Hypoplastic right heart syndrome *Hypoplastic left heart syndrome *Coarctation of the aorta

Hypoplastic left heart syndrome

A group of anomalies characterized by a small or absent right ventricle is: *Turner syndrome *Hypoplastic right heart syndrome *Hypoplastic left heart syndrome *Coarctation of the aorta

Hypoplastic right heart syndrome

A reduction in the distance b/n the orbits is referred to as: Anophthalmia Micrognathia Hypertelorism Hypotelorism

Hypotelorism

Inter hemispheric fissure

Hypoxia Groove within the midline of the brain that divides the two cerebral hemispheres

Treatment of adenomyosis

Hysterectomy or hormone therapy

Ovarian dysgenesis

Imperfect or abnormal development of the ovaries

Hypoplasia

Incomplete growth of a structure or organ

PID is a common cause of ?

Infertility

Fetal presentation is determined upon the fetal part closest to the: A. External os of the cervix B. Fundus of the uterus C. Internal os of the cervix D. body of the uterus

Internal os of the cervix

Parts of the fallopian tube

Interstitial - The segment of the fallopian tube that lies within the uterine horn Isthmus - short and narrow segment of the fallopian tube Ampulla - the longest and most tortuous segment of the fallopian tube; area of tube where fertilization takes place and a common location for ectopic pregnancies to implant. Infundibulum - The distal segment of the fallopian tube Fimbria - fingerlig extensions of the fallopian tube

What pregnancies are considered potentially life-threatening because the pregnancy May progress normally until spontaneous rupture occurs

Interstitial pregnancies

What are the sonography findings of a krukenberg; sertoli leydig; dysgerminoma; yolk sac?

Krukenberg- bilateral, smooth walled hypoechoic ovarian Mass; may have ascites Dysgerminoma- Ovoid, solid echogenic Mass; may contain some cystic components Sertoli Leydig/yolk sac - Varying sonography appearances

Fetal lung maturity can be assessed using the: *Lecithin to sphingomyelin ratio *Systolic to diastolic ratios *Estriol to alpha-fetoprotein ratio *Lung size formula

Lecithin to sphingomyelin ratio

An echogenic intracardiac focus is most often seen w/in the: Right atrium Left atrium Right ventricle Left ventricle

Left ventricle

Tetralogy of Fallot consists of all of the following except: *Overriding aortic root *Ventricular septal defect *Pulmonsary stenosis *Left ventricular hypertrophy

Left ventricular hypertrophy

Palpable abdominal mass, Dysuria and Urinary fequency GYN diagnosis

Leiomyoma Leiomyosarcoma

Palpable adnexal mass GYN diagnosis

Leiomyoma (pedunculated) Ovarian Mass PID

The difference between leiomyoma and leiomyosarcoma

Leiomyoma is benign Leiomyosarcoma is malignant, rapidly growing and most commonly found with perimenopausal or postmenopausal women.

In VACTERL association, the letter "L" stands for: Limb Lung Liver Larynx

Limb

Sonographic findings of different IUDs

Lippes Loop - 5 equally spaced shadowing structures Dalkon Shield - shadowing ovoid shape device

The lack of sulk within the fetal cerebrum is a reliable indicator of: A. agenesis of the corpus callosum B. lissencephaly C. schizencephaly D. porencephaly

Lissencephaly

Which form of holoprosencephaly consistent with life.

Lobar

All of the following are midline brain structures except: A. interhemispheric fissure B. cavum septum pellucidum C. lobes of the thalamus D. third ventricle

Lobes of the thalamus

Focal myometrial contraction

Localized painless contractions of the myometrium in the gravid uterus that should resolve within 20 to 30 minutes

Fetal lie can be described as

Long or trv

Ectopic pregnancy labs

Low HCG Low Hematocrit after rupture

Down syndrome (T21) labs

Low HCG Low MSAFP Low Estriol Low PAPP-A High Inhibin A

Edwards Syndrome and Turner syndrome (T18) labs

Low HCG Low MSAFP Low Estriol Low PAPP-A Low Inhibin A

For gestational trophoblastic disease what are the most common metastatic involvement?

Lungs, liver, vagina

Ovary and endometrial phase day 15-28

Luteal Phase (OV) Secretory phase (Endo)

What is particularly helpful in triple screen for neural tube defects?

MSAFP

An unusual protuberance of the tongue is termed: Epignathus Macrognathia Pharyngoglossia Macroglossia

Macroglossia

Which of the following laboratory values would be significant in the detection of an abdominal wall defect? *Maternal serum alpha-fetoprotein *Human chorionic gonadotropin *Maternal serum amylase *Estradiol

Maternal serum alpha-fetoprotein

Double-layer thickness

Measurement of the endometrium from basal layer to basal layer excluding both the adjacent hypoechoic Myometrium and the intracavitary fluid(if present)

Cephaloceles are common findings in

Meckel gruber syndrome

The syndrome associated w/ an occipital cephalocele, cystic renal disease, and polydactyly is: Meckel-Gruber syndrome Potter syndrome VACTERL association Sirenomelia syndrome

Meckel-Gruber syndrome

An omphalocele is associated w/ all of the following except: *Pentalogy of Cantrell *Trisomy 18 *Patau syndrome *Meconium aspiration syndrome

Meconium aspiration syndrome

The "lying down" adrenal sign would be seen in all of the following situations except: *Unilateral renal agenesis *Bilateral renal agenesis *Potter syndrome *Megacystis

Megacystis

What's the term for enlargement of the urinary bladder? Posterior urethral valves Urethral atresia Prune belly syndrome Megacystis

Megacystis

Types of cephalocele

Meningocele - meninges only Encephalocele - brain tissue only Encephalomeningocele - both meninges and brain tissue Encephalomeningocystocele - Menignes, brain tissue, and lateral ventricle

Congenital uterine malformations is aka

Mullerian anomalies

Numerous noncommunicating anechoic masses are noted w/in the left renal fossa of a fetus at 20 wks gestation. What's the most likely etiology of these masses? *Autosomal recessive polycystic kidney disease *Autosomal dominant polycystic kidney disease *Multicystic dysplastic kidney disease *Hydronephrosis

Multicystic dysplastic kidney disease

All of the following are associated w/ gastroschisis except: *Normal cord insertion *Multiple chromosomal abnormalities *Elevated maternal serum alpha-fetoprotein *Periumbilical mass

Multiple chromosomal abnormalities

All of the following are characteristic sonographic findings of achondrogenesis except: *Micromelia *Absent mineralization of the pelvis *Multiple dislocated joints *Polyhydramnios

Multiple dislocated joints

Arachnoid granulations

Nodular structures located along the falx cerebri that reabsorb cerebrospinal fluid into the venous system; aka arachnoid villi

All of the following are characteristics of spina bifida cystica except: Banana sign Lemon sign Enlarged mass intermedia Normal MSAFP

Normal MSAFP

All of the following are associated w/ omphalocele except: Normal cord insertion Multiple chromosomal abnormalities Elevated MSAFP Periumbilical mass

Normal cord insertion

The anechoic space along the posterior aspect of the fetal neck is the: A. Nuchal fold B. Nuchal cord C. Nuchal Translucency D. Rhombencephalon

Nuchal translucency

How is OFD and HC meas.

OFD - outer of occipital bone to outer frontal bone. HC - outer perimetar of skull (cand be obtained with OFD help)

All of the following are associated w/ esophageal atresia except: Down syndrome VACTERL syndrome Edwards syndrome Oligohydramnios

Oligohydramnios

OEIS complex

Omphalocele, Bladder Extrophy, Imperforate anus, Spinal bifida, associated with cloacal extrophy

A bell-shaped chest and multiple fetal fractures are indicative of: Thanatophoric dysplasia Cuadal regression syndrome Achondrogenesis Osteogenesis imperfecta

Osteogenesis imperfecta

Upon sonographic interrogation of a 28-wk pregnancy, you note that when pressure is applied to the fetal skull, the skull can be easily distorted. This is sonographic evidence of: Arnold-Chiari II malformation Achondroplasia Thanatophoric dysplasia Osteogenesis imperfecta

Osteogenesis imperfecta

What are the ovaries consist of?

Outer cortex and inner medulla.

What are the clinical findings of an ovarian dysgerminoma

Ovarian Dysgerminoma 1. younger than 30 2. ^ lactate dehydrogenase 3. ^HCG sometimes 4. Precocious puberty 5. may be found during preganancy

Oliguria and Ovarian enlargement GYN diagnosis

Ovarian hyperstimulation syndrome (cystic enlargement)

N/V GYN diagnosis

Ovarian hypersyimulation syndrome Ovarian torsion

Acute pelvic pain GYN diagnosis

Ovarian mucinous cystadenocarcinoma Ovarian mucinous cystadenoma Ovarian serous cystadenocarcinoma Ovarian serous cystadenoma Ovarian Torsion Pelvic inflammatory disease Ruptured ovarian hemorrhagic cyst Perforated intrauterine contraceptive device

Elevated AFP GYN diagnosis

Ovarian yolk sac tumor

Endometriosis is most commonly located where?

Ovaries

Fallopian tubes AKA

Oviducts, uterine tubes, salpiges

What is cited with the most common cause for Androgen Excess?

PCOS

Hirsutism GYN diagnosis

PCOS Sertoli-Leydig cell tumor (androblastoma)

IUD has been linked with

PID Ectopic pregnancy spontaneous abortions

Mittelschmertz

Pain at the time of ovulation

What is the classic clinical Triad of an ectopic pregnancy?

Pain, vaginal bleeding, palpable Mass

Cerebral peduncles

Paired structures located anterior to the cerebral aqueduct

Dermoid plug

Part of a dermoid tumor that contains various tissues and may produce the posterior shadowing during a sonography exam

Pathologies with High PAPP-A

Patau Syndrome (Trisomy 13)

Fusion of the orbits and holoprosencephaly are associated with: A. Edwards syndrome B. Turner syndrome C.Down syndrome D. Patau syndrome

Patau syndrome

Which of the following is also referred to as trisomy 13? A. Down Syndrome B. Edwards Syndrome C. Turner Syndrome D. Patau Syndrome

Patau syndrome

PID may manifest after?

Pelvic Surgery TB Association of Appendix Ruptured Colonic Diverticulum

Inferlility GYN diagnosis

Pelvic inflammatory disease (chronic)

Before 9 wks, the fetal kidneys are located w/in the: A. Renal fossae B. Pelvis C. Chest D. Umbilical cord

Pelvis

Duodenal atresia and esophageal atresia are associated w/: Oligohydramnios Polyhydramnios Normal amniotic fluid index Both A and B

Polyhydramnios

Following an intracranial hemorrhage, a cyst is noted within the cerebrum that communicates with the lateral ventricle. This is referred to as: A. schizencephaly B. lissencephaly C. holoprosencephaly D. porencephaly

Porencephaly

Location of piriformis

Posterior

Which artifact listed below is produced when the sound beam is barely attenuated through a fluid or a fluid-containing structure? A. Reverberation artifact B. comet tail artifact C. posterior shadowing D. posterior enhancement

Posterior enhancement

Which artifact listed below is cause by attenuation of the sound beam? A. Reverberation artifact B. comet tail artifact C. posterior shadowing D. posterior enhancement

Posterior shadowing

Where is the ovarian fossa located?

Posterior to the ureter and internal iliac artery and superior to the external iliac artery

Preeclampsia

Pregnancy induced maternal high blood pressure and excess protein in the urine after 20 weeks gestation

The quadruple screen includes an analysis of all of the following except: A. HCG B. AFP C. Inhibin A D. Pregnancy-associated plasma protein A

Pregnancy-associated plasma protein A

The brain is separated into 3 primary vesicles termed.

Prosencephalon (forebrain) Mesencephalon (midbrain) Rhombencephalon (Hindbrain)

What is the primary purpose of the fallopian tube?

Provide an area for fertilization and to offer a means of transportation for the products of conception to reach the uterine cavity

Malignant ovarian tumors may leak mucinous material, a condition known as: A. Dandy-walker syndrome B. Pseudomyxoma peritonei C. Asherman syndrome D. Fitz-Hugh-Curtis syndrome

Pseudomyxoma peritonei

Difference between endometriosis and adenomyosis

Pt's with endometriosis tend to be younger and have fertility troubles pt's with adenomyosis are older and multiparous

Precocious puberty is best defined as: A. Pubertal development before the age of 8 B. Pubertal development before the age of 13 C. Excessive hair growth in girls in areas where hair growth is normally negligible D. changes within the female that are caused by increased levels of AFP

Pubertal development before the age of 8

A separate mass of nonfunctioning fetal lung tissue is referred to as: *Pulmonary adenomatoid malformation *Pulmonary sequestration *Cystic adenomatoid malformation *Bat wing sign

Pulmonary sequestration

Which CIV is shorter and more vertical than the other

RCIV is shorter and more vertical than the left.

Absence of the radius is referred to as: Talipes equinovarus Clubfoot Radial ray defect Phocomelia

Radial ray defect

Pelvic muscles

Rectus Abdominis iliopsoas obturator internus (anterolateral wall) piriformis (posterolateral wall) pelvic diaphragm (levator ani and coccygeus)

What are the functions of estrogen?

Regeneration of endometrium Induction of salt and water retension Stimulates contractions

What are the ovaries responsible for?

Releasing estrogen and progesterone throughout the menstrual cycle

Failure of the kidneys to form is called: A. Hydronephrosis B. Renal dysplasia C. Renal agenesis D. Renal ectopia

Renal agenesis

The "lying down" adrenal sign describes the sonographic findings of: *Enlarged bladder and urethra *Renal agenesis *Multicystic dysplastic kidney disease *Posterior urethral valves

Renal agenesis

Obstruction at the level of the ureteropelvic junction would lead to dilation of the: *Renal pelvis and bladder *Bladder and ureter *Ureter and renal pelvis *Renal pelvis and calices

Renal pelvis and calices

Uterine malformations often lead to what?

Repeated abortions

What age are dermoids commonly found in?

Reproductive age group but may also be found in the postmenopausal patients

Endometrioma are found in whom?

Reproductive years

What do dermoids result from?

Retention of an unfertilized ovum that differentiates into the three germ cell layers

Which of the following would be caused by a large acoustic interface and subsequent production of false echoes? A. Posterior shadowing B. Acoustic enhancement C. Mirror image D. reverberation

Reverberation

The most common fetal cardiac tumor is the: Rhabdomyoma Chordae tendineae Cardiomyoma Cystic adenomatoid malformation

Rhabdomyoma

Gastroschisis occurs more often in what location? *Left lateral of the cord insertion *Right lateral of the cord insertion *Just superior to the fetal bladder *In the base of the umbilical cord

Right lateral of the cord insertion

Right ovarian vein vs left ovarian vein

Right ovarian vein drains into IVC Left ovarian vein drains into left renal vein

The optimal scan plane to visualize micrognathia is: Transverse Axial Sagittal Coronal

Sagittal

A large space between the first and second toe is termed A. Polydactyly B. Club foot C. Ulnaration D. Sandal gap

Sandal Gap

Why is PID linked to Ectopic Pregnancy?

Scar formation can disrupt the motility and function of the tube and inhibit the likelihood of conception

The development of fluid-filled cleft within the cerebrum is consistent with: A. holoprosencephaly B. lissencephaly C. schizencephaly D. hydranencephaly

Schizencephaly

What are some causes of Female Infertility ?

Septate Uterus Endometriosis Luteal phase deficiency

What is the most common malignancy of the ovary?

Serous cystadenocarcinoma

Together what comprise most neoplasms of the ovary?

Serous cystadenoma and cystic teratoma

What are the clinical findings of a Sertoli leydig tumor?

Sertoli leydig 1. younger than 30 2. abnormal menstruation 3. hirsutism 4. Virilization

Virilization GYN diagnosis

Sertoli-Leydig cell tumor (androblastoma) ovarian carcinoma.

Rhizomelia denotes: *Long upper extremeties *Shortening of an entire limb *Shortening of the proximal segment of a limb *Shortening of the distal segment of a limb

Shortening of the proximal segment of a limb

What condition is associated w/ bilateral renal agenesis, oligohydramnios, and fusion of the lower extremities? Sacrococcygeal teratoma Caudal displacement syndrome Sirenomelia Osteogenesis imperfecta

Sirenomelia

Lissencephaly

Smooth brain condition where there is little to no gyri or sulci within the cerebral cortex

What is the sonography appearance of Fallopian tube carcinoma?

Solid Mass within the adnexa, tube may become distended secondary to obstruction or infection, fluid contained within the distended tubes could be simple serous fluid, blood, or pus

Adnexal ring sign

Sonographic sign which describes the appearance of an ectopic pregnancy within the fallopian tube

A myelomeningocele is associated with: A. Down syndrome B. Spina Bifida C. Edwards syndrome D. Patau Syndrome

Spina bifida

Stages of ovarian carcinoma

Stage I- Tumor is confined to the ovary Stage 2- Tumor involves one or both ovaries with pelvic extension Stage 3- Tumor involves one or both ovaries with confirmed peritoneal Mets outside of the pelvis and or Regional lymph node involvement Stage 4- Distant Mets behind the peritoneal cavity

All of the following are sonographic findings of Arnold-Chiari 2 malformation except: A. Enlarged massa intermedia B. hydrocephalus C. obliteration of the cisterna magna D. strawberry sign

Strawberry sign

Cumulus oophorus

Structure that contains the developing oocyte

Sulci vs gyri

Sulci -Grooves within the brain Gyri - Folds in the cerebal cortex

Pts in their late 2nd or 3rd trimester may suffer from

Supine hypotensive syndrome

The reduction in blood return to the maternal heart caused by the gravid uterus compressing the maternal inferior vena cava describes: A. Edwards syndrome B. pulmonary obstructive syndrome C. supine hypotensive syndrome D. Recumbent hypotensive syndrome

Supine hypotensive syndrome

Webbed fingers or toes are termed : A. Clinodactyly B. Syndactyly C. Polydactyly D. Whren syndrome

Syndactyly

All of the following are associated w/ amniotic band syndrome except: Amputation of fetal parts Anencephaly Facial clefting Synechiae

Synechiae

Anopthalmia is linked with

T13 T18

radial ray defect can be seen in what abnormalities

T13 T18 other syndromes

Macrognathia is associated with

T13 and T18 and other chromosomal syndromes

T18 vs T21 vs T13 vs T8 vs Turner syndrome

T18/T21/Patau - Chromosomal aberration in which there is a third chromosome T18 - AKA Edwards syndrome T-21 - AKA Down syndrome T13 - AKA Patau syndrome often associated with holoprosencephaly T8 - AKA warkany syndrome Turner - A chromosomal aberration where one sex chromosome is absent AKA Monosomy x

The most common abnormalities associated with increased NT is

T21 T18 Turner syndrome Congestive Heart failure

A microcephalic headshape is associated with

TORCH infections Trisomy 13 and 18 Meckel-Gruber syndrome Fetal alcohol syndrome

The cerebellum is measured in the

TRV plane at the same level as the cisterna magna and thalamus.

A cloverleaf skull and hydrocephalus is seen w/: Achondrogenesis Osteogenesis imperfecta Sirenomelia Thanatophoric dysplasia

Thanatophoric dysplasia

A cloverleaf-shaped skull is related to: A. T18 B. Meckel-gruber syndrome C. T13 D. Thanatophoric dysplasia

Thanatophoric dysplasia

Aqueduct stenosis

The abnormal narrowing of the cerebral aqueduct

Nuchal translucency

The anechoic space along the posterior aspect of the fetal neck

Which of the following best describes transposition of the great vessels? *The aorta arises from the left ventricle and the pulmonary artery arises from the right ventricle. *The aorta arises from the right ventricle and the pulmonary artery arises from the left ventricle. *The aortic arch is narrowed and positioned anterior to the pulmonary vein. *The presence of an omphalocele and ectopic cordis.

The aorta arises from the right ventricle and the pulmonary artery arises from the left ventricle.

Intradecidual sign

The appearance of a small gestational sac in the uterine cavity surrounded by the thickened echogenic endometrium

Pseudogestational sac

The appearance of an abnormally shaped false gestational Sac within the uterine cavity as a result of an ectopic pregnancy this often corresponds with accumulation of blood and secretions within the uterine cavity

Subependymal layer

The area just beneath the ependymal lining and lateral ventricles

Zygote

The cell formed by the union of two gametes, the first stage of fertilized ovum

Trophoblastic cells

The cells that surround the gestation that produce human chorionic gonadotropin

Chromosomes

The cellular structures that contain genes

Which of the following is a true statement about the fetal heart? *The apex of the heart will be angled to the right of the midline. *The apex of the heart is positioned closest to the spine. *The normal fetal heart will fill approximately 2/3 of the fetal chest. *The chamber closest to the fetal spine is the left atrium.

The chamber closest to the fetal spine is the left atrium.

the gest sac consists of 2 cavities

The chorionic cavity and the amnionic cavity.

Agenesis of corpus callosum

The congenital absence of corpus callosum which may be partial or complete

Agenesis of the corpus callosum

The congenital absence of corpus callosum which may be partial or complete

Corpus luteum of pregnancy

The corpus luteum that is maintained during an early pregnancy for the purpose of producing estrogen and progesterone; keeps endo thick

Meninges

The coverings of the brain and spinal cord Which include: Dura Mater _outer Arachnoid - middle Pia Mater - inner

Oogenesis

The creation of an ovum

Embryonic demise (fetal demise)

The death of an embryo before 10 weeks gestation

Decidua parietalis (vera)

The decidualized tissue along the uterine cavity adjacent to the decidua basalis.

Endometrial atrophy

The degeneration of the endometrium with advancing age, most often seen in postmenopausal women

Decidua basalis

The endometrial tissue at the implantation site and the maternal contribution of the placenta

Facies

The features or appearance of the face

Meningocele

The herniation of the cranial or spinal meninges due to an open cranial or spinal defect

Luteinizing hormone (LH)

The hormone of the anterior pituitary gland that surges around day 14 of the menstrual cycle, resulting in ovulation

Amnion

The inner sac which contains the embryo and embryonic fluid, echogenic curvilinear structure that may be seen during the first trimester within in the gestational sac

The abdominal circumference should include all of the following except: A. The fetal stomach B. The fetal thoracic spine C. The umbilical vein D. The kidneys

The kidneys

Glomus

The largest part of the choroid plexus

Cisterna magna

The largest sister in the school located in the posterior portion of the skull

Brain stem

The lower part of the brain composed of the pons, midbrain, medulla oblongata

Interthalamic adhesion

The mass of tissue located in the third ventricle within the midline of the brain which connects the two lobes of the thalamus AKA Massa intermedia

Mean Sac diameter

The measurement of the gestational Sac to obtain a gestational age achieved by adding the measurements of the length, width, height of gestational sac and dividing by 3

Hormone replacement therapy

The medical treatment used to accommodate the reduction of estrogen and progesterone that occurs during menopause -to prevent symptoms such as hot flash and vaginal atrophy

Which of the following isn't a true statement about the normal fetal heart? *The ventricular septum should be uninterrupted and of equal thickness to the left ventricular wall. *There is a normal opening w/in the atrial septum. *B/n the right ventricle and right atrium, one should visualize the tricuspid valve. *The mitral valve is positioned closer to the cardiac apex than the tricuspid valve.

The mitral valve is positioned closer to the cardiac apex than the tricuspid valve.

Hydatidiform mole

The most common form of gestational trophoblastic disease in which there is an excessive growth of the placenta and high levels of human chorionic gonadotropin, typically benign

Thanatophoric dysplasia

The most common lethal skeletal dysplasia characterized by a Cloverleaf skull with frontal bossing and hydrocephalus

Double decidual sign

The normal sonography appearance of the decidua capsularis and decidua parietalis separated by the anechoic fluid-filled uterine cavity, AKA double Sac sign

Non communicating hydrocephalus

The obstruction of cerebrospinal fluid from a source within the ventricular system

Foramen magnum

The opening in the base of the skull through which the spinal cord exits

Decidual reaction

The physiologic effect on the endometrium in the presence of pregnancy

Cerebellar vermis

The portion of the cerebellum located within the midline of the brain that connects it's two hemispheres

Decidua capsularis

The portion of the decidua opposite the uterine cavity, across from the decidua basalis.

Blastocyst

The stage at which the conceptus implants within the decidualized endometrium

Secondary yolk sac

The structure responsible for early nutrient transfer to the embryo, the yolk Sac seen during a sonography examination of the early gestation

Vitelline duct

The structure that connects the developing embryo to the secondary yolk sac

Polypectomy

The surgical removal of a polyp

The interthalmic adhesion (massa intermedia) passes through the: A. Third ventricle B. fourth ventricle C. cisterna magna D. cavum septum pellucidum

Third ventricle

Types of miscarriages/abortion

Threatened abortion complete (spontaneous abortion) Incomplete abortion Missed abortion Inevitable abortion

The most common sonographic appearance of pulmonary sequestration is a/an: *Dilated pulmonary artery and hypoechoic chest mass *Pleural effusion and ipsilateral hiatal hernia *Triangular, echogenic mass w/in the chest *Anechoic mass w/in the chest

Triangular, echogenic mass w/in the chest

A fetus with a karyotype revealing it has 69 chromosomes and sonographic findings of webbed fingers and intrauterine growth restriction most likely has : A. Trisomy 21 B. Trisomy 18 C. Triploidy D. Turner syndrome

Triploidy

A molar pregnancy, omphalocele, and small, low-set ears are found most often with: A.Trisomy 21 B. Trisomy 18 C. Trisomy 13 D. Triploidy

Triploidy

The most frequently encountered chromosomal abnormality associated w/ holoprosencephaly is: Triploidy Trisomy 21 Trisomy 18 Trisomy 13

Trisomy 13

What chromosomal aberration is most often associated with holoprosencephaly? A. Anopthalmia B. T21 C. T13 D. T18

Trisomy 13

Which of the following is also referred to as Patau syndrome? Trisomy 18 Trisomy 21 Trisomy 12 Trisomy 13

Trisomy 13

A strawberry-shaped skull is commonly associated with: A. T21 B. T15 C. T18 D. T13

Trisomy 18

Nonimmune hydrops and ovarian dysgenisis are found in fetuses affected by: A. Trisomy 21 B.Trisomy 18 C.Triploidy D. Turner syndrome

Turner syndrome

Webbing of the neck and short stature is found in infertile female patients with a history of: A. trisomy 21 B. Triploidy c. trisomy 13 d. Turner syndrome

Turner syndrome

4 types of osteogenesis imperfecta

Type II - a uniformely fatal and most severe form Type I, III, IV - diagnosed after birth

heterozygous achondroplasia

Type of dwarfism that is an autosomal dominant disorder that is the most common nonlethal skeletal dysplasia that is characterized by rhizomelia

If pregnancy is located within the cornea or cervix what can be done to destroy the pregnancy?

Ultrasound-guided injection of potassium chloride into the embryo or gestational sac

Cryptorchidism describes: Bilateral pelvic kidneys Urethral atresia Undescended testicles Ovarian dysgenesis

Undescended testicles

All of the following would be associated w/ oligohydramnios except: *Bilateral multicystic dysplastic kidney disease *Unilateral renal agenesis *Bilateral renal agenesis *Autosomal recessive polycystic kidney disease

Unilateral renal agenesis

Which of the following would result in compensatory hypertrophy? *Unilateral renal agenesis *Bilateral renal agenesis *Pelvic kidney *Horseshoe kidneys

Unilateral renal agenesis

An obstruction at the ureterovesicular junction would lead to dilation of the: A. Bladder and urethra B. Bladder, urethra, and ureters C. Bladder, urethra, ureters, and renal collecting system D. Ureters and renal collecting system

Ureters and renal collecting system

Placenta previa

When the placenta covers or nearly covers the internal OS of the cervix

Where can ednometriosis be located?

Within C - Section Scars, Liver, Lungs and extremities

Where is the ovum contained?

Within the cumulus oophorus. Looks like a daughter cyst

Can fibroids effect fertility?

Yes especially on location such as intracavitary or submucosal It also effects contractile motion of the UT which leads to interference with sperm migration Fibroids may prevent cervical dilatation thus needing a c-section.

What are the clinical findings of a yolk sac tumor?

Yolk sac tumor 1. younger than 20 2. poor prognosis 3. rapid growth 4. ^MSAFP

chorioangioma

a benign placental tumor

leiomyoma (Fibroid) (uterine myoma)

a benign, smooth muscle tumor of the uterus

Thalamus

a brain structure that allows communication between the senses as well as performing many other functions

nuchal fold

a collection of solid tissue at the back of the fetal neck

bicornuate uterus

a common uterine anomaly in which the endometrium divides into two horns (endometrial cavities) with a prominent concavity of the uterus

ethmocephaly

a condition in which there is no nose and a proboscis separating two close-set orbits; associated with holoprosencephaly

achondroplasia

a disorder that results in abnormal bone growth and dwarfism

A cephalic index of <75 denotes

a dolichocephalic shape

obstructive cystic dysplasia

a fetal disorder caused by early renal obstruction; leads to small and echogenic kidneys that have cysts along their margins

multicystic dysplastic kidney disease

a fetal renal disease thought to be caused byan early obstruction;leads to the development of multiple noncommunicating cysts of varying sizes in the renal fossa

adenomyoma

a focal mass of adenomyosis

Sacrococcygeal teratoma

a germ cell tumor that is the most common congenital neoplasm and more frequently found in females. Has malignant potential if large.

osteogenesis imperfecta

a group of disorders that result in multiple fractures in utero; caused by decreased mineralization and poor ossification of the bones. aka brittle disease.

limb-body wall complex is AKA

a group of disorders with sonographic findings Aka body stalk anomaly

neuroblastoma

a malignant tumor that can occur within the adrenal gland and anywhere within the sympathetic nervous system

cystic hygroma

a mass, typically found in the neck region, that is the result of an abnormal accumulation of lymphatic fluid within the soft tissue

Tuburous sclerosis

a systemic disorder that leads to the development of tumors within various organs

endoscopic-guided laser photocoagulation

a treatment that uses lasers to separate abnormal placental vascular connections between twins that are suffering from twin-twin transfusion syndrome

lambda sign

a triangular extension of the placenta at the base of the membrane is indicative of a dichorionic diamniotic pregnancy

trident hand

a wide separation between the middle and ring finger

6. What is the stage of the conceptus that implants within the decidualized endometrium? a. Blastocyst b. Morula c. Zygote d. Ovum

a. Blastocyst

25. The most malignant form of gestational trophoblastic disease is: a. Choriocarcinoma b. Hydatidiform mole c. Anembryonic d. Hydropic villi

a. Choriocarcinoma

10. In the first trimester, normal hCG levels will: a. Double every 48 hours b. Triple every 24 hours c. Double every 24 hours d. Double every 12 hours

a. Double every 48 hours

19. The most common cause of pelvic pain with pregnancy is: a. Ectopic pregnancy b. Heterotopic pregnancy c. Missed abortion d. Molar pregnancy

a. Ectopic pregnancy

1. What is the name of the dominant follicle prior to ovulation? a. Graafian b. Corpus luteum c. Morula d. Corpus albicans

a. Graafian

16. The herniation of the embryologic bowel into the base of the umbilical cord at 9 weeks is referred to as: a. Physiologic bowel herniation b. Pseudo-omphalocele c. Omphalocele d. Gastroschisis

a. Physiologic bowel herniation

The uterine position in which the corpus tilts forward and comes in contact with the cervix describes: a. anteflexion b. anteversion c. retroflexion d. retroversion

a. anteflexion

The most common uterine anomaly is the: a. bicornis unicollis b. bicornis bicollis c. uterus didelphys d. septate uterus

a. bicornis unicollis

The largest part of the uterus is the: a. corpus b. isthmus c. cervix d. fundus

a. corpus

The term that indicates the presence of two separate placentas is: a. dichorionic b. bichorionic c. monoamniotic d. diamniotic

a. dichorionic

The placenta releases __________ to maintain the corpus luteum. A. HCG B. FSH C. LH D. Gonadotropin-stimulating hormone

a. hCG

The most common form of monozygotic twins is: a. monochorionic diamniotic b. dichorionic monoamniotic c. monochorionic monoamniotic d. none of the above

a. monochorionic diamniotic

Twins that have the threat of being conjoined are: a. monochorionic monoamniotic b. monochorionic diamniotic c. dizygotic d. dichorionic diamniotic

a. monochorionic monoamniotic

Identical twins result from: a. monozygotic twinning b. dizygotic twinning c. heterotopic pregnancies d. monochorionic pregnancies

a. monozygotic twinning

All of the following are clinical findings associated with leiomyoma except: a. myometrial cysts b. infertility c. palpable pelvic mass d. menorrhagia

a. myometrial cysts

Conjoined twins that are attached at the abdomen are referred to as: a. omphalopagus b. thoracopagus c. ileopagus d. craniopagus

a. omphalopagus

The shunting of blood from one twin to the other is termed: a. twin-twin transfusion syndrome b. twin embolization syndrome c. twin peak sign d. conjoined twins

a. twin-twin transfusion syndrome

tracheoesophageal fistula

abnormal connection between esophagus and trachea

marginal cord

abnormal cord insertion at the edge of the placenta

kyphosis

abnormal curvature of the spine

sonographic appearance of kyphosis

abnormal posterior curvature of the spine.

hypospadias

abnormal ventral curvature of the penis as the result of a shortened urethra that exits on the ventral penile shaft

transposition of the great vessels

abnormality in which the aorta arises from the right ventricle and the pulmonary artery arises from the left ventricle. pulmonary artery and aorta will be straight instead of criss-cross

aortic atresia

abnormality in which there is a small or absent opening between the left ventricle and aorta

phocomelia

absent long bones with the hand and feet arise from the shoulders and hips.

placenta accreta/increta/percreta

accreta - the abnormal adherence of the placenta to the myometrium in an area where the decidua is either absent or minimal increta - invasion of the placenta within the myometrium Percreta - penetration of the placenta through the uterine serosa and possibly into adjacent pelvic organs

The horns correlate with

adjacent cranial bones

BPD,HC can be taken after how many weeks

after 1st trim between 13-14 weeks

when does the secretory phase occur

after ovulation and is stimulated by progesterone.

The size and shape of the uterus depends on

age parity presence of pathology congenital anomalies

The size of the Ovary depends on the

age of the pt

Types of holoprosencephaly

alobar semilobar lobar

parasitic twin

also known as acardiac twin

twin peak sign

also known as the lambda sign

the chorionic cavity lies between the

amnion and chorion

limb reduction is often caused by

amniotic band syndrome

Amniotic bands vs synechiae

amniotic bands are rarely seen with sonography while synichiae are seen as linear, thin meambranes with a broad base crossing the amniotic sac.

ureterocele

an abnormality in which the distal ureter projects into the urinary bladder

acardiac twin

an abnormally developed twin that has an absent upper body and no heart

circumvallate placenta

an abnormally shaped placenta caused by the membranes inserting inward from the edge of the placenta, producing a curled-up placental shape

Pituitary gland

an endocrine gland located within the brain that consists of anterior and posterior lobe.

autosomal dominant polycystic kidney disease

an inherited disease that results in the development of renal, liver, and pancreatic cysts late in life; also referred to as adult polycystic kidney disease

infantile polycystic kidney disease

an inherited renal disease that results in bilateral enlargement of the fetal kidneys and microscopic renal cysts; akaautosomal recessive polycystic kidney disease

autosomal recessive polycystic kidney disease

an inherited renal disease that results in bilateral enlargement of the fetal kidneys and microscopic renal cysts; also referred to as infantile polycystic kidney disease

pseudomyxoma peritonei

an intraperitoneal extension of mucin-secreting cells that result from the rupture of a malignant mucinous ovarian tumor or possibly a malignant tumor of the appendix.

ureterovesicular junction obstruction

an obstruction located in the region where the ureter meets the bladder

sacral dimple

an opening in the skin over the distal spine

foramen of Bochdalek

an opening located in the left posterolateral portion of the diaphragm

foramen of morhagni

an opening located right anteromedially within the diaphragm

foramen ovale

an opening within the fetal heart within the atrial septum that allows blood to flow from the right atrium to the left atrium

epignathus

an oral teratoma

ovum

an unfertilized egg

macroglossia

an unusual protuberance of the tongue

incomplete/partial fusion or agenesis of mullerian ducts will result in

anatomic variants of the uterus, cx, and/or vagina

Cavum septum pellucidum will appear as an

anechoic box-shaped structure in the axial plane.

The 4th ventricle is located

anterior to the cerebellum within the midline of the brain.

Branches of the uterine artery are

arcuate arteries radial arteries radial arteries then divide into straight arteries and spiral arteries.

isthmus

area of the uterus between the corpus and the cervix

Cornua (uterus)

areas just inferior to the fundus of the uterus where the fallopian tubes attach bilaterally

radial arteries

arteries that supply blood to deeper layers of myometrium

NT can resolve how?

as pregnancy progresses

Meigs syndrome

ascites and pleural effusion in the presence of some benign ovarian tumors

discordant fetal growth

asymmetrical fetal weight between twins

How to measure NF

axial plane at the level of the cavum septum pellucidum and angling coronally to include the cerebellum and occipital bone

11. Compared with a normal IUP, the ectopic pregnancy will have a: a. High hCG b. Low hCG c. Markedly elevated hCG d. High AFP

b. Low hCG

27. All of the following would be associated with a lower-than-normal human chorionic gonadotropin level except: a. Ectopic pregnancy b. Molar pregnancy c. Blighted ovum d. Spontaneous abortion

b. Molar pregnancy

9. What hormone, produced by the corpus luteum, maintains the thickened endometrium? a. Estrogen b. Progesterone c. Human chorionic gonadotropin d. Luteinizing hormone

b. Progesterone

18. All of the following are associated with an abnormal nuchal translucency except: a. Trisomy 21 b. Trisomy 16 c. Trisomy 18 d. Turner syndrome

b. Trisomy 16

4. The structure created by the union of sperm and egg is the: a. Blastocyst b. Zygote c. Morula d. Ampulla

b. Zygote

The invasion of endometrial tissue into the myometrium of the uterus is referred to as: a. amenorrhea b. adenomyosis c. endometriosis d. adenomyomatosis

b. adenomyosis

The outer membrane surrounding the fetus is referred to as the : a. placenta b. chorion c. amnion d. yolk sac

b. chorion

Twins whose bodies are connected at some point are said to be: a. fraternal b. conjoined c. identical d. stuck

b. conjoined

Twins that result from the fertilization of two separate ova are called: a. diamniotic b. dizygotic c. monozygotic d. monochorionic

b. dizygotic

Fraternal twins result from: a. monozygotic twinning b. dizygotic twinning c. heterotopic pregnancies d. monochorionic pregnancies

b. dizygotic twinning

The parasitic twin in acardiac twinning is also referred to as the "pump twin." a. true b. false

b. false

a 24 year old female patient presents to the emergency department for a pelvic sonogram with an indication of pelvic pain. Upon ultrasound interrogation, the monographer notes an anechoic mass within the vagina. The mass most likely represents a: a. nabothian cyst b. gartner duct cyst c. dandy walker cyst d. ovarian cyst

b. gartner duct cyst

The demise of a twin during the second or third trimester can lead to: a. twin-twin transfusion syndrome b. twin embolization syndrome c. twin peak sign d. acardiac twinning

b. twin embolization syndrome

Which form on monozygotic twinning is least common? a. monochorionic diamniotic b. monochorionic monoamniotic c. dichorionic diamniotic d. dichorionic biamniotic

b.monochorionic monoamniotic

Why does AFP stand out in neural tube defects.

because AFP exits the fetus through an opening in the neural tube if one is present, such as with open spina bifida which allows AFP to be part of the maternal circulation. But it does not solely elevated with spina bifida.

Menopausal UT

becomes smaller

Boundaries of the pelvis

begins at the iliac crest and ends at the symphysis pubis Lateral - fused ilium and ischium Posterior - sacrum, coccyx Anterior - symphysis pubis Anterolateral - hip bone, obterator internus muscles Posterolateral - piriformis, coccygeous

Reproductive hormones and how it works

beneath the thalamus

Location of aqueduct of Sylvus

between 3rd and 4th ventricle

When does the axial skeleton begin to form?

between 6-8th week

The body of the lat vent is located

between the frontal and occipital horns

cloacal extrophy

birth defect consisting of omphalocele, bladder extrophy, imperforate anus, spina bifida, also referred to as OEIS complex

The internal iliac arteries supply:

bladder uterus vagina rectum

The morula continues to differentiate and form a structure called a

blastocyst

What happens in days 20 and 21 of the menstrual cycle?

blastocysts begins to implant into the decidualized endometrium

hematocolpos

blood accumulating in the vagina

hematometra

blood accumulating within the uterine cavity

hematometrocolpos

blood accumulation within the uterus and vagina

Hydranencephaly vs alobar holoprosencephaly, vs hydrocephaly

both hydrocephalus and holoprosencephaly will have a rim of cerebral tissue where as hydranencephaly there is no cerebral mantle present. Hydranencephaly is also fatal.

homozygous achondroplasia occurs when

both parents are dwarfs

The paired lateral ventricles are located on

both sides of the falx cerebri within the cerebral hemispheres.

The ovaries get blood supply from

both the ovarian artery and uterine artery.

The anterior fontanelle, when completely filled with bone, is referred to as

bregma

Obstructive hydrocephalus

buildup of CSF within the ventricular system secondary to some type of obstruction.

Coronary Heart disease

buildup of placque within arteries that supply the myocardium of the heart

How can HCG be detected?

by blood (quantative and qual) and urine (qual)

2. Fertilization typically occurs within ____ after ovulation. a. 40 hours b. 12 hours c. 24 hours d. 56 hours

c. 24 hours

23. All of the following are sonographic findings consistent with ectopic pregnancy except: a. Decidual thickening b. Complex free fluid within the pelvis c. Bilateral, multiloculated ovarian cysts d. Complex adnexal mass separate from the ipsilateral ovary

c. Bilateral, multiloculated ovarian cysts

12. The first sonographically identifiable sign of pregnancy is the: a. Amnion b. Yolk sac c. Decidual reaction d. Chorionic cavity

c. Decidual reaction

15. During a first-trimester ultrasound examination, you note a cystic structure within the fetal head. This most likely represents the: a. Prosencephalon b. Mesencephalon c. Rhombencephalon d. Proencephalon

c. Rhombencephalon

26. A sonographic examination was performed on a pregnancy patient who complained of vaginal bleeding. Sonographically, a crescent- shaped anechoic area is noted adjacent to the gestational sac. The gestational sac contained a 6-week single live intrauterine pregnancy. What is the most likely diagnosis? a. Ectopic pregnancy b. Molar pregnancy c. Subchorionic hemorrhage d. Anembryonic gestation

c. Subchorionic hemorrhage

8. What structure lies within the extraembryonic coelom? a. Gestational sac b. Embryo c. Yolk sac d. Amnion

c. Yolk sac

Acardiac twinning result from: a. poor maternal nutrition b. dizygotic gestations c. abnormal links among placental vessels d. twin embolization syndrome

c. abnormal links among placental vessels

A succenturiate lobe of the placenta refers to as: a. bilobed placental lobe b. circumvallate placental lobe c. accessory lobe d. circummarginate placental lobe

c. accessory lobe

The inner membrane surrounding the fetus is referred to as: a. placenta b. chorion c. amnion d. yolk sac

c. amnion

Which term relates the number of amniotic sacs? a. chorionicity b. placentation c. amnionicity d. both a. and b.

c. amnionicity

The rigid region of the uterus located between the vagina and the isthmus is the: a. cornu b. corpus c. cervix d. fundus

c. cervix

The maternal contribution to the placenta is the: a. chorionic vera b. decidua vera c. decidua basalis d. chorion frondosum

c. decidua basalis

Twins having two placentas and two amniotic sacs are referred to as: a. monochorionic diamniotic b. biamniotic dichorionic c. dichorionic diamniotic d. dichorionic biamniotic

c. dichorionic diamniotic

Factors that increase the likelihood of having multiple gestations include all of the following except: a. advanced maternal age b. ovulation induction drugs c. poor nutritional state d. maternal predisposition for twins

c. poor nutritional state

Leiomyosarcoma of the uterus denotes: a. The benign invasion of endometrial tissue into the myometrium b. the ectopic location of endometrial tissue in the adnexa c. the malignant counterpart of a fibroid d. an anechoic, simple cyst located within the cervix

c. the malignant counterpart of a fibroid

Congenital malformation of the uterus that results in complete duplication of the genital tract is: a. Unicornate uterus b. bicornis bicollis c. uterus didelphys d. supseptate uterus

c. uterus didelphys

How to meas lateral vent

calipers are placed at the level of the glomus of the choroid plexus.

Types of neural tube defects

cephaloceles anencephaly spina bifida

The thalamus should not be confused with

cerebral peduncles which are more inferiorly positioned in the brain.

What is the largest part of the brain

cerebrum

The cerebrum can be divided into 2 main parts

cerebrum and cerebellum

the cardinal ligament supports

cervix - extends from the lateral surface of the cx to the lateral fornix of vagina

Uterine artery runs medially off the internal iliac artery towards the _____________ where it then ascends lateral to the uterus in the broad ligament to the cornua. There it courses _______________ toward the ovary to where it anastomoses with a branch of the ovarian artery

cervix; laterally

virilization

changes within the female that are caused by increased androgens; may lead to deepening of the voice and hirsutism.

Endometriomas are also called

chocolate cysts

The blastocyst makes links with the endometrium via small projections of tissue called

chorionic villi

double bubble sign

classic sono sign of duodenal atresia representing the stomach and proximal duodenum

What is the most common congenital abnormalities

cleft lip and or palatte

Ovulation induction treatments are

clomid pergonal

spina bifida occulta signs

closed spinal lesions (aka closed spina bifida) - completely covered by skin - difficult to identify sonographically Postnatally - sacral dimple - hemangioma - lipoma - excessive hair over the distal spine.

heterotopic pregnancy

co-existing ectopic and intrauterine pregnancies

monozygotic

coming from one fertilized ovum or zygote

septate uterus

common congenital malformation of the uterus hat results in a single septum with completely separate two endometrial cavities.

What are the 2 main types of hydrocephalus

communicating and noncommunicating

What happens on day 28?

complete implantation where both fetus and mother are connected.

erythroblastosis fetalis

condition in which there is an incompatibility between the fetal and maternal red blood cells

anemia

condition of having a deficient number of red blood cells

nuchal cord

condition of having the umbilical cord wrapped completely around the fetal neck

anorectal atresia

congenital maldevelopment of the rectum and absence of the anal opening causing dilation of bowel

subseptate uterus

congenital malformation of the uterus that results in a normal uterine contour with an endometrium that branches into two horns with normal uterine fundus

unicornuate uterus

congenital malformation of the uterus that results in a uterus with one horn

uterus didelphys

congenital malformation of the uterus that results in the complete duplication of the uterus, cervix and vagina

The most common cause of hydrocephalus in utero

congenital obstruction of the ventricular system by means of aqueductal stenosis.

Vein of Galen aneurysm is associated with

congestive heart failure

Each fallopian tubes attaches to the uterus at the level of the uterine horns called the

cornua

What type of imaging of the face has been shown very effective in discovering cleft lip/pallate?

coronal and axial

After corpus luteum it then turns into

corpus albicans which is seen as a small echogenic scar on the ovary.

The cavum septum pellucidum is located inferior to

corpus callosum

The cerebral hemispheres are linked in the midline by the

corpus callosum

A brachycephalic headshape is associated with

craniosyntosis Trisomy 21 and 18

The primary vesicles form the

critical brain structures

The uterus can be divided into 4 sections

cx, isthmus, corpus, fundus.

What is the most common in utero infection?

cytomegalovirus

14. The normal gestational sac will grow: a. 2 mm per day b. 3 mm per day c. 1 cm per day d. 1 mm per day

d. 1 mm per day

21. All of the following are contributing factors for an ectopic pregnancy except: a. Pelvic inflammatory disease b. Assisted reproductive therapy c. Intrauterine contraceptive device d. Advanced paternal age

d. Advanced paternal age

20. The most common location of an ectopic pregnancy is the: a. Ovary b. Interstitial portion of the uterine tube c. Cornual portion of the uterine tube d. Ampullary portion of the uterine tube

d. Ampullary portion of the uterine tube

All of the following are sonographic findings consistent with adenomyosis except: a. Diffuse, enlarged uterus b. Myometrial cysts c. Hypoechoic areas adjacent to the endometrium d. Complex adnexal mass

d. Complex adnexal mass

17. The most common pelvic mass associated with pregnancy is the: a. Uterine leiomyoma b. Dermoid cyst c. Theca luteum cyst d. Corpus luteum cyst

d. Corpus luteum cyst

30. All of the following may be sonographic findings in the presence of an ectopic pregnancy except: a. Pseudogestational sac b. Corpus luteum cyst c. Adnexal ring d. Double sac sign

d. Double sac sign

13. The first definitive sonographic sign of an intrauterine pregnancy is the recognition of a/an: a. Yolk sac b. Embryo c. Decidual reaction d. Gestational sac

d. Gestational sac

7. Another name for the chorionic sac is the: a. Chorionic cavity b. Extraembryonic coelom c. Amniotic sac d. Gestational sac

d. Gestational sac

5. The trophoblastic cells produce: a. Estrogen b. Progesterone c. Follicle-stimulating hormone d. Human chorionic gonadotropin

d. Human chorionic gonadotropin

24. All of the following are consistent with a hydatidiform mole except: a. Heterogenous mass within the endometrium b. Bilateral theca lutein cysts c. Hyperemesis gravidarum d. Low hCG

d. Low hCG

29. Which of the following locations is most likely with metastatic gestational trophoblastic disease? a. Rectum b. Brain c. Spleen d. Lungs

d. Lungs

28. All of the following are clinical findings consistent with a molar pregnancy except: a. Vaginal bleeding b. Hypertension c. Uterine enlargement d. Small for dates

d. Small for dates

Which of the following can occur as a result of monozygotic twinning? a. monochorionic diamniotic twins b. monochorionic monoamniotic twins c. dichorionic diamniotic twins d. all of the above

d. all of the above

An anechoic mass is noted within the umbilical cord during a routine sonographic examination. What is the most likely diagnosis? a. hemangioma b. vasa previa c. chorioangioma d. allantoic cyst

d. allantoic cyst

The non functional outer layer of the endometrium is the: a. myometrium b. endometrial cavity c. functional layer d. basal layer

d. basal layer

Which term relates the number of placentas? a. chorionicity b. placentation c. amnionicity d. both A and B

d. both A and B

The paired embryonic ducts that develop into the female urogenital tract are the: a. fallopian ducts b. wolffian ducts c. gartner ducts d. mullerian ducts

d. mullerian ducts

The early gest sac looks like

decidualized (thickened) endo with gest sac called interdecidual sign

Typically, with a miscarriage, the serum human chorionic gonadotropin value will be: A. Elevated C. Decreased B. This laboratory finding is not helpful D. unchanged

decreased

dysplasia

denotes the abnormal development of a structure

Germ cell tumor

derived from germ cells of the gonads, may be found outside the reproductive tract.

Uterus and kidneys during embryologic development....

develop at the same time which means if something is wrong with the uterus, something might be wrong with the kidneys so check the kidneys.

anasarca

diffuse edema

caliectasis

dilation of the calices

pelviectasis

dilation of the renal pelvis; may also be referred to as pyelectasis

The most common location of spina bifida

distal lumbosacral region

Double decidual signs is aka

double sac sign

diethylstilbestrol

drug given to women from 1940-1970s to treat abortions and ended up giving female children malformations of the uterus

Normal cerebellum looks

dumbell shaped or figure 8 in posterior cranium

moiety

duplex collecting system

The proliferative phase can be divided into

early and late proliferative

Sonographic appearance of corpus callosum

echogenic band of tissue within the midline of the brain connecting the 2 cerebral hemispheres.

How does the spine look like with spina bifida?

echogenic laminae are normally angled inward while with spina bifida, the defective laminae will be angled outward or be said to splay.

What does the interhemispheric fissure look like?

echogenic linear formation in the midline of the fetal brain.

Assistive reproductive therapy are at increased risk for

ectopic pregnancy, heterotopic pregnancy, multiple gestations, and ovarian hyperstimulation syndrome.

The inner part of the blastocyst will develop into the

embryo, amnion, umbilical cord, and the primary and secondary yolk sacs.

What kind of glands are the ovaries exocrine or endocrine?

endocrine

Between the 2 layers of the endo lies the

endometrial (uterine) cavity which is continuous with the fallpian tubes laterally and cx inferiorly

What is the most common cause of postmenopausal bleeding (PMB)?

endometrial atrophy

Tamoxifen is linked with

endometrial hyperplasia endometrial ca endo polyps Produces a more heterogeneous and thickened endometrium appearance.

compensatory hypertrophy

enlargement of an organ secondary to an increased workload; often seen when part of an organ has been destroyed or when there is absence or decreased function of paired organs

What can cause ovarian torsion?

enlargement of ovary such as with ovarian hyperstimulation syndrome.

IUD's create posterior shadowing that is called

entrance and exit echo

Each ventricle is lined by a membrane called

ependyma

Epignathus vs microglossia sonographic appearance

epignathus is most liekly complex while microglossia is completely solid

What stimulates the endometrium to thicken?

estrogen

The endometrium is influnced by what hormones?

estrogen and progesterone

Fibroid grown is associated with

estrogen stimulation therefore it increases in size with pregnancy and decreses after menopause

space of retzius (retropubic or prevesical space)

extraperitoneal space between the bladder and symphysis pubis that contains fat

HCG levels in molar pregnancy

extremely high

Agenesis

failure of an organ to grow during embryologic development

Anopthalmia cause

failure of the optic vessel to form

Clomid or clomiphene citrate

fertility drug used to treat anovulation

in vitro fertilization

fertility treatment that requires that a mature ovum be extracted from the ovary with fertilization taking place outside the body

Conception is aka

fertilization

nonstress test is aka

fetal cardiotocography

biparietal diameter

fetal head measurement obtained in the transverse plane at the level of the third ventricle and thalamus

immune hydrops

fetal hydrops due to Rh incompatbility

ductus arteriosus

fetal shunt that connects the pulmonary artery to the aortic arch

ductus venosus

fetal shunt that connects the umbilical vein to the IVC

meconium

fetal stool that is composed of fetal skin, hair, amniotic fluid, and bile

neonatal

first four weeks after birth

amniotic sac

fluid-filled space, created by the amnion, surrounding the developing embryo or fetus

The ovarian cycle consists of 2 phases

follicular phase and luteal phase

What are the 6 cerebral lobes?

fontal lobe 2 temporal lobes 2 parietal lobes occipital lobe

placentation

formation or structure of a placenta

Chorion frondosum vs chorionic laeve

frondosum- The part of the chorion covered by chorionic villi that is the fetal contribution of the placenta laeve - the portion of the chorion that does not contain chorionic villi

Each lateral ventricle consist of

frontal, temporal, and occipital horn.

Which layer of the endo is shed during menstration?

functional layer which is why this layer thickens

Wharton's jelly

gelatinous material that is located within the umbilical cord around the umbilical vessels

Where is the origin of an intracranial hemorrhage?

germinal matrix

A dermoid or tematoma can consist of things like

glandular thyroid components, bone, hair, sebum, fat, cartilage,teeth and digestive elements

Abnormal NT meas

greater than 3mm

The resistance pattern of the Middle cerebral artery should be

greater than that of the umbilical artery and thus should be compared when fetal shunting is suspected

amniotic band syndrome

group of abnormalities associated with the entrapment of fetal parts in the amnion, often resulting in fetal amputations or clefting

pentalogy of Cantrell

group of anomalies that include an omphalocele, ectopic cordis, cleft sternum, anterior diaphragmatic defect, pericardial defects

pelvic diaphragm

group of pelvic muscles consisting of the levator ani and and coccyges muscles that provide support to the pelvic organs

cotyeldons

groups or lobes of chorionic villi

monoamniotic

having one amniotic sac

monochorionic

having one chorion

monochorionic monoamniotic

having one placenta and one amniotic sac

monochorionic diamniotic

having one placenta and two amniotic sacs

diamniotic

having two amniotic sacs

dichorionic

having two chorionic sacs

dichorionic diamnionic

having two placentas and two amniotic sacs

menometrorrhagia

heavy and prolonged bleeding between periods

Bochdalek hernia

herniation through the left posterolateral portion of diaphragm, most common

HCG levels in twin preg

high

T13 (patau syndrome) is present in 50-70% of fetuses diagnosed with

holoprosencephaly

Cleft lip/palatte is associated with

holoprosencephaly T13 amniotic band syndrome most cases are not associated with anything

The most common cause of hypotelorism

holoprosencephaly by T13 (most common cause of holoprosencephaly and hypotelorism)

HCG (human chorionic gonadotropin)

hormone produced by trophoblastic cells of the early placenta; also a tumor marker of non gravid patients and males.

Treatment for fibroids

hormone therapy

The divisions of the lateral ventricles are called

horns

GTD types

hydatidaform mole invasive mole choriocarcinoma

Ventriculomegaly vs Hydrocephalus

hydrocephalus is for severe cases of ventriculomegaly and are caused by some type of obstruction to the flow of CSF.

Fetal sutures are seen sonographically as

hypoechoic spaces between the bones

The internal iliac arteries are AKA

hypogastric arteries

The innominate bones consist of

ilium (lateral), ischium (lateral), and pubic symphysis (anterior)

linea terminalis (iliopectineal line)

imaginary line that separates the true pelvis from the false pelvis

implantantaion of blastocyst in endometrium can sometimes cause

implantation bleeding

A subchorionic hemorrhage results from

implantation of the fertalized ovum into the uterus with subsequent low-pressure bleeding.

The amnion and chorion typically fuse when

in the middle of the first trimester but may not be tottally fused until 16 weeks.

The most common location for a cephalocele is

in the occipittal region

Adenomyosis is most commonlyfound where?

in the posterior portion of the UT

The lateral ventricle is measured

in the transaxial plane at the level of the atrium.

appendicular skeleton

includes the bones of the upper extremities, lower extremities, and pelvic girdle

hyperemic

increase in blood flow

A cystic hygroma should not be confused with

increased nuchal translucency or nuchal fold thickening.

Endometrial polyps is linked with

infertility

Pergonal

infertility medicine used to stimulate follicular development of the ovaries

gamete intrafallopian tube transfer

infertility treatment in which oocytes and sperm are placed in the fallopian tube by means of laparoscopy

cholangitis

inflammation of the bile ducts

The vagina is composed of 3 layers

inner mucosal middle muscular outer (adventitia)

The cerebrum can be divided into right and left hemisphere by the

interhemispheric fissure

Hypertelorisom is diagnosed with which diameter

interlocular diameter

What is the most common finding in premature infants weighing less that 1500g and born before 32 weeks?

intracranial hemorrhage

Are the ovaries intraperitoneal or retroperitoneal?

intraperitoneal

posterior urethral valves

irregular thin membranes of tissues located within the male posterior urethra that do not allow urine to exit the urethra

If fertilization occurs what happens to the corpus luteum?

it is maintained and becomes the corpus luteum of pregnancy.

If fertilization does not occur, what happens to the corpus luteum?

it regresses and becomes the corpus abicans.

eventration of the diaphragm

lack of muscle in the dome of the diaphragm

The posterior fontanelle, when completely filled with bone, is referred to as

lambda

What is the most common benign GYN tumor?

leiomyoma (fibroid)

How to get femur length

long axis of femoral shaft U/S beam perpendicular to shaft.

The false pelvis contains

loops of bowel

HCG levels in ectopic preg

low

What passes through the 3rd ventricle to connect the two lobes of the thalamus?

massa intermedia (interthalmic adhesion)

Causes of neural tube defects

maternal diabetes valproic acid (seizure med) T18 T13 triploidy

pre gestational diabetes

maternal diabetes that existed before pregnancy

What is a predisposing condition of intracranial hemorrhage in utero?

maternal platelet disorders (most common) cocaine trauma amniocentesis

Ovarian arteries/veins course

medial (arteries) within the suspensory ligaments

Cisterna magna meas that is >10mm is consistent with

mega cisterna magna dandy walker complex

Mega cisterna magna vs dandy walker malformation

mega cisterna magna is an enlarged cisterna magna without involvement of fourth ventricle whereas dandy walker involves the fourth ventricle.

selective reduction

method of reducing the number of pregnancies in a multiple gestation whereby certain embryos/ fetuses are terminated

Patau Syndrome (T13) labs

mildly high MSAFP

Cryptomenorrhea

monthly symptoms ofmenstruation without bleeding.

rhabdomyoma

most common fetal heart tumor, echogenic mass within the myocardium

spina bifida aperta

most common form of spina bifida; open lesions AKA spina bifida cystica/spinal dysrapism

The lat vent that will be readily seen on sono is

most often the vent farthest from transducer

Myometrium

muscular layer that has the bulk of the uterine tissue. Where contractile motion occurs

hydro/metrocolpos is usually seen in

neonates and have palpable masses

The neural plate will give rise to the

neural tube

Spina bifida

neural tube defect that occurs when the embryonal neural tube fails to close.

WHat is seen in the neck with fetuses with downs syndrome in 2nd trim?

nuchal thickening edema redundant skin

What are the 3 meas that can be obtained in the TRV plane of the fetal face at the level of the eyes?

occular diameter interlocular diameter binocular diameter

Elevated AFP is linked with

omphalocele gastroschisis multiple gestations fetal demise incorrect gestational dating

The 2 lobes of the thalamus are located

on both sides of the third ventricle

Each fetal vertebrae consists of 3 echogenic ossification centers

one centrum and 2 neural processes

Sonographic appearance of acrania/ anencephaly

one of the most common neural tube defects 1. Absent cranial vault 2. some cerebral tissue may be present 3. "froglike" facies or bulging eyes

The broad ligament is seen

only when there is free fluid

What is the most common form of spina bifida?

open spina bifida or spina bifida aperta/cystica

Chorion

outer membrane of a gestation that surrounds the amnion and developing embryo

NF calipers is placed

outer to outer

The uterine wall consists of 3 layers

outermost layer - serosal layer (perimetrium) middle layer - myometrium inner layer - endometrium

1st trimester miscarriages are linked with

ovarian abnormalities aneuploid fetuses maternal infections physical abuse trauma drug abuse maternal endocrine abnormalities anatomic factors

Proliferative phase lasts until

ovulation

What shape is the uterus?

pair shaped

papillary muscle

paired muscles in both sides of heart that hold in place either the mitral or tricuspid valves

coccygeus

pelvis muscle located posteriorly within the pelvis that helps support the sacrum

Arcuate arteries

peripheral arteries of the uterus that lie at the edge of the myometrium

hepatorenal space

peritoneal space located between the liver and right kidney AKA morisons pouch

tubal ligation (tubil sterilization)

permanent form of female sterilization in which the fallopian tubes are severed

Corpus luteum cyst

physiologic cyst that develops after ovulation has occurred

The outer part of the blastocysts, tophoblastic tissue, forms into the

placenta and chorion.

In 2nd trimester painless vaginal bleeding is most often associated with

placenta previa

bilobed placenta

placenta that consists of two separate discs of equal size

In 2nd trimester painful vaginal bleeding is most often associated with

placental abruption

abruptio placentae

placental abruption

The fluid in the peritoneal cavity usually releases where?

posterior cul-de-sac

Where is the cerebellum located?

posterior fossa of the cranium

Internal iliac arteries location

posterior to UT and OV and enters the true pelvis near the sacral prominence

string of pearls sign

presence of 10 or more small cysts measuring 2-18mm along the periphery of the ovary

uterine artery embolization

procedure used to block the blood supply to a leiomyoma

During the second half of the menstrual cycle (luteal phase), the corpus luteum produces

progesterone and in small amounts estrogen

During the second half of the menstral cycle, following ovulation what is produced?

progesterone by the corpus luteum of the ovary

vernix

protective fetal skin covering (greasy covering during birth)

cephalocele

protrusions of intracranial contents through a defect in the skull

the interdecidual sign (double decidual sac) can resemble a

pseudogestational sac of an ectopic pregnancy

hemato/metrocolpos is usually seen with

pubescent teens. Frequently associated with imperforate hymen

Pyometra

pus within uterus

radial ray defect

rare absence or underdevelopment of the radius

vaginal fornices

recesses of the vagina

hypoterlorism

reduced distance between the orbits

amnionicity

relates to the number of amnions in a multiple gestation

chorionicity

relates to the number of chorions and the type of placentation in a multiple gestation

zygosity

relates to the number of zygotes (fertilized ova)

What is the function of the uterus?

reproduction

Ovarian hyperstimulation syndrome

resulting from hyperstimulation of the ovaries by fertility drugs

Is the uterus intraperitoneal or retroperitoneal?

retroperitoneal

Comet Tail

reverberation artifact cause by small, highly reflective interfaces such as gas bubbles

The bony pelvis consists of

sacrum, coccyx, and innominate bones. These are the bones that mark the pelvic cavity.

pseudoprecocious puberty

secondary sexual development induced by sex steroids or from other sources such as ovarian tumors

decidual reaction (sign) also looks like

secretory phase

pulmonary sequestration

separate mass of nonfunctioning lung tissue with its own blood supply, triangle echogenic mass

What are the clinical findings of serous cystadenoma vs mucinous cystadenoma and carcinomas?

serous - Asymptomatic Mucinous - pelvic pressure and swelling Both Carcinomas 1. Weight loss 2. pelvic pressure and swelling 3. abnormal vaginal bleeding 4. GI symptoms 5. acute abdominal pain associated with torsion or rupture 5.elevated ca-125

What are the sonography findings of serous cystadenoma/carcinoma; mucinous cystadenoadenoma/carcinoma?

serous cystadenoma- 1.Large, multilocular and or papillary projections 2. Bilateral 3. carcinoma- same but with ascites Mucinous cystadenoma - 1. same as serous but may contain debris 2. usually unilateral 3. carcinoma - same but with pseudomyxoma peritonei (complex ascites)

Chamydia

sexually transmitted disease that can lead to inflammatory disease in both sexes

gonorrhea

sexually transmitted disease that can lead to pelvic inflammatory disease

Micromelia

shortening of an entire limb

Acromelia

shortening of the distal segment of a limb

mesomelia

shortening of the middle segment of a limb

twin-twin transfusion syndrome

shunting of venous or arterial blood from one twin to another through placental circulation

The neural tube will become the

spine and the brain

Krukenberg tumor is a malignant ovarian tumor that metastasized from the GI tract most frequently the what?

stomach

straight arteries/spiral arteries

straight - supplies blood to the basal layer of the endometrium spiral - supplies blood to the functional layer of the endo

BPP criteria

stress - 2 fetal hrt accelerations (more than 15bpm and more than 15 sec) together with one fetal movt Thoracic movements - breathing lasting 30 sec Fetal movts - 3 movts Fetal tone - one flexion or extension of limb AFI - At least 1 SDP meas 2cm.

Potter syndrome

syndrome characterized by bilateral renal agenesis, abnormal facies, pulmonary hypoplasia, limb abnormalities

Prune belly syndrome

syndrome that is a consequence of the abdominal wall musculature being stretched by an extremely enlarged urinary bladder

progestin

synthetic progesterone secreted by some intrauterine devices to regulate menstrual flow

Clinical findings of Supine hypotensive syndrome

tachycardia/sweating nausea pallor

follicular aspiration

technique used for in vitro fertilization in which follicles are drained for oocyte retrieval

assisted reproductive therapy

techniques used to treat infertitlity

chordae tendineae

tendons within the heart that attach the tricuspid valve in the right ventricle and the mitral valve in the left ventricle to their respective papillary muscle

What is the most common intracranial tumor in utero?

teratoma

cleft palate

the abnormal development of the soft and/or hard palate of the mouth where there is a division in palate

velamentous cord insertion

the abnormal insertion of the umbilical cord into the membranes beyond the placental edge

Colpocephaly

the abnormal lateral ventricle shape in which there is a small frontal horn and enlarged occipital horn

cloverleaf skull

the abnormal shape of the cranium caused by premature fusion of the sutures in which there is frontal bossing and a cloverleaf shape to the skull

Pulmonary atresia

the absence of the pulmonary valve, which in turn prohibits blood flow from the right ventricle into the pulmonary artery and essentially to the lungs

frontal bossing

the angling of the frontal bones that produces an unusually prominent forehead

hemivertebra

the anomaly of the spine in which there is absence of all or part of the cerebral body and posterior element

zygote

the cell formed by the union of two gametes; the first stage of a fertilized ovum

the yolk sac is located within

the chorionic cavity between the amnion and chorion

vanishing twin

the death and reabsorption of a twin

fetus papyraceus

the death of one fetus in a twin pregnancy that is maintained throughout the pregnancy; actually means paper-like fetus

The placenta is formed by

the decidua basalis and chorion frondosum

Hematopoiesis

the development of blood cells

Morula

the developmental stage of the conceptus following the zygote

cleavage

the division of a cell

WIthin the atria of both lateral ventricles lies

the echogenic configuration of the choroid plexus. Choroid plexus may be also found in the roof of the third and fourth ventricles.

cloaca

the embryonic structure that develops into the rectum and urogenital sinus

homozygous achondroplasia

the fatal form of achondroplasia

neonatal period

the first 28 days of life

clubfoot is aka

the foot is most often inverted and rotated medially. aka talipes equinovarus

The rhombancephalon will develop into

the fourth ventricle and several other essential brain structures.

The higher the location of spina bifida,

the greater the neurologic involvement

diaphragmatic hernia

the herniation of the abdominal contents into the chest cavity through a defect in the diaphragm

ureteropelvic junction

the junction of the ureter and renal pelvis

ureterovesicular junction

the junction of the ureter and the urinary bladder

The neural processes of each vertebra will become

the lamina, pedicle, TRV process, spinous process, and articular process.

Tricuspid regurgitation

the leakage of blood back through the tricuspid valve

interocular diameter

the length between the orbits; measured from the medial margin of one orbit to the medial margin of the other orbit

levoverted uterus

the long axis of the uterus deviating to the left of midline

extroverted uterus

the long axis of the uterus deviating to the right of the midline

ebstein anomaly

the malformation or malpositioning of the tricuspid valve that causes multiple heart defects

The pituitary gland is referred to as

the master gland

ocular diameter

the measurement from the lateral margin of the orbit to the medial margin of the same orbit

Choriocarcinoma

the most malignant form of gestational trophoblastic disease with possible mets to the liver, lungs and vagina.

Alobar holoprosencephaly

the most severe form of holoprosencephaly

fimbrae ovarica

the one fimbrae attached to the ovary

perimetrium

the outer layer of the uterus aka serosal layer

serosal layer

the outermost layer of the uterus

cervical incompetence

the painless dilation of the cervix in the second or early third trimester

cerclage

the placement of sutures within the cervix to keep it closed

Trigone or atrium of the lateral ventricle

the point at which the body, temporal horn, and occipital horn meet

cerebellum

the portion of the brain located in the inferior posterior part of the skull that is responsible for motor output, sensory perception, and equilibrium

nuchal

the posterior part or nape of the neck

mortality

the rate of actual deaths

morbidity

the relative frequency of occurence of a disease

The anterior lobe of the pituitary is responsible for

the release of two chief hormones that influence the menstrual cycle: FSH LH

oocyte retrieval

the removal of oocytes from ovarian follicles by aspiration

funneling (cervical)

the result of the premature opening of the internal os and the subsequent bulging of the membranes into the dilated cervix

sticky bands result from

the rupture of the amnion and can entrap fetal parts and cause amputation of digits, limbs, and even the skull. and can also lead to peculiar facial clefting.

premature rupture of membranes

the rupture of the amniotic sac prior to the onset of labor

"keyhole sign"

the sonographic appearance of a dilated fetal bladder and urethra in the presence of a bladder outlet obstruction

"lying down" adrenal sign

the sonographic appearance of the adrenal gland in a parallel position within the abdomen as a result of renal agenesis

banana sign

the sonographic sign of the cerebellum being curved in the presence of spina bifida

Vertebral column

the structure that runs the length of the spine and contains the spinal cord.

lower uterine segment

the term used for the isthmus of the uterus during pregnancy

Each lateral ventricle communicates with

the third ventricle in the midline of the brain at the foramen of Monro, or the paired interventricular foramina.

anteversion

the typical version of the uterus where the uterine body tilts forward forming a 90 degree angle with the cervix

retroflexion

the uterine body tilts backward and comes in contact with the cervix, forming an acute angle between body and cervix

The centrum will form

the vertebral body

The yolk sac is connected to the embryo by

the vitelline duct (omphalomesenteric duct) which contains one artery and one vein

AFP is produced respectively by

the yolk sac, fetal gastrointestinal tract, and the fetal liver.

Estrogen is first produced by

theca internal cells of the secondary follicles during the first part of the menstrual cycle.

What happens in the first half of the menstra cycle to the endo

thickening due to estrogen

The amnion looks like

thin, echogenic line loosely surrounding embryo

How to get Head circumference/bpd

third ventricle thalamus cavum septum pellucidum falx cerebri

postpartum

time directly after giving birth and extending to about 6 weeks

What is the primary role of the fimbria?

to draw the unfertalized egg into the tube.

The purpose of BPP is

to investigate for signs of fetal hypoxia

What is the primary responsiility of the hypothalamus as it relates to the menstrual cycle?

to regulate the release of hormones by the anterior pituitary gland.

Maternal serum screening is aka

triple screen

Gestational trophoblastic disease GTD results in excessive growth of

trophoblastic cells

The outer layer of the blastocyst is called

trophoblastic cells

syncytiotrophoblastic cells

trophoblastic cells surrounding the blastocyst that produce hCG

fetal circulation

umbilical vein - half goes to the liver, half goes to IVC via the ductus venosus IVC - right atrium - foramen ovale - left atrium - left ventricle - aorta OR IVC - right atrium - right ventricle - pulmonary artery - lungs OR right ventricle - ductus arteriosus - descending aorta

Pulmonary hypoplasia

underdevelopment of the lungs

Where are the cerebellar tonsils located?

undersurface of the cerebellum and become distorted with spina bifida and arnold chiari malformations

Cleft lip can be what types

unilateral, bilateral, or midline

The ovaries form and where and descend to the pelvis in utero?

upper abdomen

The internal iliac arteries give rise to the

uterine arteries

corpus

uterine body

The uterine arteries supply blood to the

uterus, fallopian tubes, and overies.

Post menopausal uterus an ovaries.

uterus-smaller but same shape OV - smalle and more echogenic, lack folicles, harder to image

tricuspid valve

valve between right atrium and right ventricle

anastomoses

vascular connections

causes for limb-body wall complex

vascular occlusion, amnion rupture, embryonic dysgenesis.

What lies between the two laminar and posterior to the centrum

vertebral column

Molar Pregnancy labs

very high MSAFP

FHR depending on weeks

weeks 5-6: 100-115bpm week 9: 140bpm 2nd trimester to term: 120-140bpm

Presentation of fetus is determined on

what is closest to the internal os of cervix

stuck twin

when a twin fetus, suffering from twin-twin transfusion syndrome, experiences severe oligohydramnios and is closely adhered to the uterine wall

retained products of conception

when additional placental tissue remains within the uterus after the bulk of the placenta has been delivered

The interthalmic adhesion part of the thalamus can be seen when

when enlarged or surrounded by CSF. It passes through the third ventricle.

luteal phase deficiency

when the endometrium does not develop appropriately in the luteal phase of the endometrial cycle as a result of reduced progesterone production

shoulder dystocia

when the shoulder of the fetus cannot pass through the birth canal during pregnancy

twin embolization syndrome

when vascular products travel from a demised twin to the surviving twin by means of the common vascular channels within the shared placenta

Can you fix spina bifida?

yes even as early as 16 weeks

Sonographic Findings of Chronic PID

1. Hydrosalpinx 2. Scars in Fallopian Tubes (Echogenic bands in tube) 3. Adhesions may obliterate distinct borders 4. Multicystic and solid complex adnexal masses (tubo-ovarian complex or abscess)

Clinical Findings of Chronic PID

1. Irregular Menses 2. Palpable Adnexal Mass 3. Infertility

Causes of postmenopausal bleeding include all of the following except: A. Asherman syndrome B. Endometrial atrophy C. Endometrial hyperplasia D. Intracavitary fibroids

A. Asherman syndrome

Fitz-Hugh-Curtis syndrome is described as: A. Clinical findings of gallbladder disease as a result of pelvic inflammatory disease B. The presence of uterine fibroids and adenomyosis in the gravid uterus C. Co-existing intrauterine and extrauterine pregnancies D. The presence of pyosalpinx, hydrosalpinx, and endometritis

A. Clinical findings of gallbladder disease as a result of pelvic inflammatory disease

Tamoxifen effects on the endometrium will sonographically appear as: A. Cystic changes within a thickened endometrium B. Cystic areas within a thin endometrium C. Thin endometrium D. No apparent effect on endometrial thickness or appearance

A. Cystic changes within a thickened endometrium

The sonographic findings of an endometrial polyp include: A. Diffuse thickening of the endometrium B. Menometrorrhagia C. Intermenstrual bleeding D. Infertility

A. Diffuse thickening of the endometrium

A 60-year-old patient presents to the emergency department with sudden onset of vaginal bleeding. The sonographic examination reveals an endometrium that measures 4 mm. There are no other significant sonographic findings. What is the most likely diagnosis? A. Endometrial atrophy B. Endometrial carcinoma C. Endometrial polyp D. Cervical stenosis

A. Endometrial atrophy

A 34-year-old patient presents to the ultrasound department for an endovaginal ultrasound complaining of intermenstrual bleeding. The sonographic findings include a focal irregularity and enlargement of one area of the endometrium. The most likely diagnosis is: A. Endometrial polyps B. Endometrial carcinoma C. Endometrial atrophy D. Intramural leiomyoma

A. Endometrial polyps

The development of adhesions between the liver and diaphragm as a result of pelvic inflammatory disease is termed: A. Fitz-Hugh-Curtis syndrome B. Dandy-Walker syndrome C. Stein-Leventhal syndrome D. Asherman syndrome

A. Fitz-Hugh-Curtis syndrome

Possible benefits of estrogen replacement therapy: A. Reduction in osteoporosis risk B. Estrogenate C. Tamoxifen D. CA-125

A. Reduction in osteoporosis risk

Hypomenorrhagia GYN diagnosis

Asherman syndrome

Amenorrhea GYN diagnosis

Asherman syndrome PCOS

Postdilitation and postcurettage GYN diagnosis

Asherman syndrome endometritis retained products of conception

An asymptomatic 65-year old patient presents to the ultrasound department with pelvic pain but no vaginal bleeding. Her endometrial thickness should not exceed: A. 6 mm B. 8 mm C. 5 mm D. 3 mm

B. 8 mm

The removal of tissue from the endometrium by scraping is termed: A. Dialation B. Curettage C. Sonohysterography D. Hysterography

B. Curettage

The most common cause of postmenopausal bleeding is: A. Endometrial carcinoma B. Endometrial atrophy C. Endometrial leiomyoma D. Cervical carcinoma

B. Endometrial atrophy

A 68-year-old patient presents to the ultrasound department complaining of vaginal bleeding. The most likely cause of her bleeding is: A. Endometrial carcinoma B. Endometrial polyps C. Endometrial atrophy D. Endometrial fibroids

C. Endometrial atrophy

Endometrial hyperplasia may be caused by all of the following except: A. Hormone replacement therapy B. Estrogen replacement therapy C. Endometrial atrophy D. Tamoxifen

C. Endometrial atrophy

The most common female genital tract malignancy is: A. Ovarian carcinoma B. Cervical carcinoma C. Endometrial carcinoma D. Pelvic inflammatory disease

C. Endometrial carcinoma

Tamoxifen has been linked with all of the following except: A. Endometrial polyps B. Endometrial hyperplasia C. Endometrial leiomyomas D. Endometrial carcinoma

C. Endometrial leiomyomas

______ is a gynecologic procedure to remove an endometrial polyp. A. Hysterectomy with myomectomy B. Histogram with myomectomy C. Hysteroscopy with polypectomy D. Hysteroscopy with polyp myectomy

C. Hysteroscopy with polypectomy

Polycystic ovarian disease is associated with: A. Fitz-Hugh-Curtis syndrome B. Plateau syndrome C. Stein-Leventhal syndrome D. Asherman syndrome

C. Stein-Leventhal syndrome

The breast cancer treatment drug that may alter the sonographic appearance of the endometrium is: A. Progesterone B. Estrogenate C. Tamoxifen D. CA-125

C. Tamoxifen

The most common initial clinical presentation of pelvic inflammatory disease: A. Endometritis B. Tubo-ovarian abscess C. Vaginitis D. Pyosalpinx

C. Vaginitis

The most common cause of pelvic inflammatory disease is: A. Intrauterine contraceptive use B. Postabortion C. Ruptured appendix D. Chlamydia

D. Chlamydia

A female patient presents to the ultrasound department with a clinical history of Clomid treatment. She is complaining of nausea, vomiting, and abdominal dissension. What circumstance is most likely causing her clinical symptoms? A. Stein-Leventhal syndrome B. Polycystic ovarian disease C. Fitz-Hugh-Curtis syndrome D. Ovarian hyperstimulation syndrome

D. Ovarian hyperstimulation syndrome

All of the following are considered risk factors for pelvic inflammatory disease except: A. Intrauterine contraceptive device B. Multiple sexual partners C. Post childbirth D. Uterine leiomyoma

D. Uterine leiomyoma

Elevated HCG on nongravid pts and Elevated lactate dehydrogenase GYN diagnosis

Ovarian dysgerminoma

Postmenopausal vaginal bleeding GYN diagnosis

Ovarian granulosa cell tumor Ovarian mucinous/serous cystadenocarcinoma/adenoma Ovarian Thecoma

The breast cancer drug that inhibits the effects of estrogen in the breast is: A. CA-125 B. Methotrexate C. RA-916 D. Tamoxifen

Tamoxifen

Which of the following definitions best describes the term adnexa? A. The area posterior to the uterus, between the uterus and rectum B. The area located posterior to the broad ligaments and adjacent to the uterus C. The area anterior to the uterus, between the uterus and urinary bladder D. The area lateral to the iliac crest and posterior to the pubic symphysis

The area located posterior to the broad ligaments and adjacent to the uterus.

Infertility is best defined as: A. The inability to conceive a child after 6 months of unprotected intercourse B. The inability to conceive a child after 3 months of unprotected intercourse C. The inability to conceive a child after 1 year of unprotected intercourse D. The inability to conceive a child after 2 years of unprotected intercourse

The inability to conceive a child after 1 year of unprotected intercourse

lysis

destruction or breaking down (i.e. hemolysis - the breaking down of blood components)

Precocious puberty is linked to

ovarian, adrenal , and liver tumors.

CT (Computed Tomography)

uses ionizing radiation


Kaugnay na mga set ng pag-aaral

Module 18. Vision: Sensory and Perceptual Processing

View Set

Abd-Chap.23-24 Prostate & scrotum NH

View Set

PSYCH 312: Human Sexuality-Sexually Transmitted Infections/Diseases

View Set

Geometry Chapter 8 Always/Sometimes/Never

View Set

econ final review lake norman charter

View Set

Counseling theories, Comprehensive exam

View Set

Chapter 5 Analyzing the Audience

View Set

Ch 2. Organizational Theories for Human Resources

View Set

OT Survey, Abraham through Joseph, Test

View Set